Download as pdf or txt
Download as pdf or txt
You are on page 1of 103

PHY 203.

ELECTRONICS

Introduction

The study of solid materials is concerned with the properties of solid has great utility
that result from the distribution of atoms and electrons in the material. A good
understanding of the crystal structure and energy band theory in the solid materials
is very crucial in understanding different properties and associated applications of
these materials. In this module we will study the:

 Lattices and crystal structures of SC, BCC, FCC and diamond, Crystal binding.
Introduction to semiconductor physics: energy bands, electrons and holes.

 Intrinsic and extrinsic semiconductors; n- and p- type semiconductors, n-p junction,


biasing of diode, diode equation, Zener-diode.

 Half wave and full wave rectifiers, bridges rectifiers, filters, ripple factor, voltage
regulation, voltage stabilization using Zener diode, p-n-p and n-p-n transistors,
transistor characteristics.

 Load lines and DC bias circuits.

 Different methods of biasing a transistor.

 Transistor equivalent circuit.

CHAPTER I: STRUCTURE OF SOLIDS AND ENERGY BAND THEORY

1. 0. STRUCTURE OF SOLIDS

 Crystal structure: Solids are composed by atoms held together by chemical


bonds. Any substance can, depending on its temperature and pressure, exists in
one of the three states of matter which are solid, liquid and gas. These states of
matter are only distinguished by differences in macroscopic properties.

However, discussions on microscopic scale reveal only two states of matter namely
ordered and disordered states. On one hand, disordered materials include gases
which are completely disordered and liquids which are disordered over large
distances but hold a short range order. The short range order means that around
any particular molecule or atom in the liquid, the position of neighboring atoms or
molecules is not perfectly random. On the other hand, ordered state includes
crystalline solids which constitute the majority of solids and are characterized by
a long range ordering.

1
Figure 1.1: An example of Crystals.

 Polycrystalline: Polycrystals formed from different single crystals (crystallites or


grains) with size ranging from nanometers to millimeter are also included in this
category. Polycrystalline are made up by single crystals but orientated in different
directions.

 Amorphous: Certain solids such as glass, polymer, rubber etc. which do not
completely exhibit the regular structure are called amorphous solids.

Fig.1.2: a) Crystalline b): Polycrystalline c): Amorphous

Examples of crystals

Fig.1.3: Fluorite: CaF2 Quartz: SiO2 Barite: BaSO4

1.1. Basic definitions.

2
In the following discussion we will be focusing on crystalline materials only. X-rays
diffraction experiments have clearly shown that atoms or molecules in a crystal are
in regular and periodic arrangement. The crystal grows as if identical building
blocks were added on continuously. Thus we will define important basic terms.

a) Crystal lattice.

The regular pattern (periodic and systematic) of points which describes the three
dimensional arrangement of atoms, ions or molecules in a crystal structure is
called the crystal lattice. It is a kind of mathematical construction representing
the crystal structure where motives are located at or near the intersection of an
imaginary grid and the motives are repeated indefinitely in 3 dimensions.

Fig.1.3: A two dimensional lattice

The lattice point may represent a single atom, ion or groups of atoms or molecules
termed as the basis or motive. Those building blocks or unit cells in a solid crystal
 
are chosen so that the parallelogram constructed on its basic vectors a1 and a2 has
the shortest possible magnitude as indicated on the above figure.

A lattice translation operation or crystal translation operation is defined as the


displacement of a crystal parallel to itself by a crystal translation vector
   
r  n1a1  n2 a2  n3a3
  
Where a1 , a2 and a3 are non-coplanar vectors which characterize the unit cell and

are called lattice vectors and ni are unit vectors, with i  1, 2,3 integers. r is called
the translation vector. It shows that the positions of all lattice points in the crystal

structure can be reached by drawing all possible vectors of the form r .
  
a1 , a2 and a3 vectors are normally taken along the three edges of the unit cell. The
angles ,  and  are angles between pair of edges namely.
 a2 , a3  ,  a3 , a1  and  a1 , a2  respectively.

3
Fig.1.4: Example of unit cell of the general shape

Identical unit cells are joined together giving rise to periodicity of lattice and the
  
basis vectors a1 , a2 and a3 are described in terms of their length also called lattice
constants a1 , a2 , a3 and the angle between them  ,  and  .

  
It should be noticed that the basis vectors a1 , a2 and a3 cannot be uniquely selected
for a given lattice points. The only requirement is that all lattice points can be
reached by a combination of the lattice vectors. The best choice is a set the shortest
possible vectors and in that case we will have a primitive unit cell.

b) Lattice planes and Miller indices

Crystal planes come from the structures known as crystal lattices. These lattices
are three dimensional patterns that consist of symmetrically organized atoms
intersecting three sets of parallel planes. These parallel planes are "crystal planes"
and are used to determine the shape and structure of the unit cell and crystal
lattice. The planes intersect with each other and make 3D shapes that have six
faces. Lattice points must be thought of as laying on various set of parallel planes
in three dimensions.

Fig. 1.5: Lattice plane in a three dimension crystal

The orientation of a crystal plane is determined by three points in the plane


provided they are not collinear. In crystallography, the orientation of a crystal

4
plane is indicated by a set of three numbers called Miller indices. The Miller
indices are obtained in the following steps:

i. Determine the intercepts of the plane along the axis X,Y, and Z in terms of
the lattice constants a, b and c.
ii. Determine the reciprocal of these numbers, 1/X, 1/Y, and 1/Z.
iii. Find the least common denominator (usually the smallest) and multiply each
fraction by this denominator.
The result is written in the form (h k l) and is called Miller indices.

Example: Let a plane have intercepts 1, 1 and 1/2 with the axis. The reciprocal are
1/1, 1/1 and 2. The common denominator is 1. Multiplying each fraction by 1 we
obtain (1; 1 ; 2) which are the Miller indices of that plane.

For intercepts at infinity the corresponding index is zero and for planes that cuts
the axis on the negative side of the origin, the corresponding index is negative and
is written with a bar above the index. Finally, we must notice that the indices (h k l)
do not define a particular plane but a set of parallel planes. Here are some
important planes in a cubic crystal:

Fig.1.6: Planes in cubic crystal

Other examples of lattice planes.

The (001), (100), (010) and (0 0-1 (-100), (0 -1 0) planes form the faces of the unit cell.
Here, they are shown as the faces of a triclinic (a ≠ b ≠ c, α ≠ β ≠ γ) unit cell .

5
The (011), (101), (110) and (0 1-1) , (1 0-1), (1-1 0) planes form the sections through
the diagonals of the unit cell, along with those planes whose indices are the
negative of these. In the image the planes are shown in a different triclinic unit cell.

To draw more other lattices you can visit the following website:
(http://www.doitpoms.ac.uk/tlplib/miller_indices/lattice_draw.php)

c) Bravais lattices.

As mentioned above the space lattice can be built up by stacking together unit cells in
perfect alignment in 3 dimensions. There are only 14 ways in which a space lattice
can be performed. Those are called 14 Bravais lattices and are grouped into 7 basic
unit cells also called seven crystal systems. Those systems are defined by the length
of lattice vectors a , b , c and the angle they are making. The following figure shows
the 14 Bravais lattices.

6
Fig.1.7: The 14 Bravais lattices

7
The following are the unit specification for each system:

System Unit cell specification

1. Cubic a  b  c and       900

2. Tetragonal a  b  c and       900

3. Orthorhombic a  b  c and       900

4. Trigonal a  b  c and       900

5. Monoclinic a  b  c and     900  c

6. Hexagonal 3 equal coplanar axes at 1200 and the 4th axis c


perpendicular to this plane a  c

7. Triclinic a  b  c and       900

Table 1.

d) The cubic system.

This is the simplest crystal classes due to the fact that the unit cell is cubic. Depending
on the arrangement of atoms in the unit cell, the cubic system can exist as: simple cubic
(SC) or primitive cell, body centred cubic (BCC) or face centred cubic (FCC).

i) The simple cubic system or primitive cell.

In the simple cubic structure, atoms are located at each corner of the unit cell. Each
lattice point is sheared between 8 similar unit cells and each unit cell has 8 lattice
points.

Fig.1.8: A unit cell of simple cubic system

The total number of lattice points per unit cell in a simple cubic system is therefore one.

Example simple cubic: Polonium is the only element that crystallise in sc structure

ii) The body centred cubic.

8
In this case atoms or lattice points are located at each corner and one atom is at the
centre of the unit cell and is not sheared by any other unit cell.

Fig.1.9: A unit cell of a body centered cubic system.

The total number of lattice points per unit cell is therefore two. The nearest atom to the
corner atom is the body centred atom. Therefore the coordination number of a bcc
structure is 8. Examples of bcc structures are barium, sodium, tungsten, uranium etc.

iii) The face centred cubic structure.

In the face central cubic structure, there are 8 atoms at the corners which are sheared
between 8 unit cells and there is an atom in the face centre which is sheared between
two unit cells. The number of atoms per unit cell is four.

Fig.1.10: A unit cell of a face centered cubic system.

Each corner atom has four neighbouring atoms in its own plane, four neighbouring
atoms in a plane above and four neighbouring atoms in a plane below it. The total
number of neighbouring atoms or coordination number is 12. Examples of FCC
structure are copper, aluminium, nickel, silver, gold, platinum etc.

e) Atomic Packing fraction for cubic structure.

The packing fraction is the fraction of volume in a crystal structure that is occupied by
constituent particles. It is a dimensionless quantity and always less than unity. It is
defined as the ratio of the volume occupied by atoms in the unit cell to the total volume.
It is written as:

where Natoms is the number of atoms in the unit cell, Vatom is the volume of an atom, and
Vunit cell is the volume occupied by the unit cell.

9
Atomic packing fraction is the ratio of the volume occupied by atoms in the unit cell to
the total volume.

Natoms is the number of atoms in the unit cell, Vatom is the volume of an atom, and Vunit cell
is the volume occupied by the unit cell.
1. Atomic packing fraction for a Simple Cubic system (SC)
Taking the edge of the cubic as a and the atomic radius as r. Assuming that atoms in
crystal structure are hard spheres, the atomic packing fraction is defined as the ratio of
the actual volume occupied by the spherical atoms in the unit cell to the total volume of
the unit cell

Fig.1.11 Atomic packing in simple cubic system

4
a Z  r3
In a simple cubic system r  and therefore f  3 ,
2 a3

where Z is the number of atoms in the unit cell. In simple cubic system with Z=1:

3
4 a
   
f  1
3  2  .
3
a 6

Therefore, f= 52.4% is the packing fraction in simple cubic system. I.e. only 52%
volume of unit cell is occupied by atoms.

2. Atomic packing fraction for Body Centred Cubic system (BCC)

Figure12: Body centered structure bcc

Because the volume of each corner atom is shared between adjacent cells, each BCC cell
contains two atoms. In this structure The number of atoms is given by:

10
8
 Z 1  2
8

Each corner atom touches the center atom. Along the diagonal we have D=4r , where r is
the radius of an atom. By geometry or Pythagoras, the length of the diagonal is D= a√3.

Therefore, the length of each side of the BCC structure can be related to the radius of
the atom by:

Knowing this and the formula for the volume of a sphere, it becomes possible to
calculate the APF as follows:

 AF= 68% which is the packing fraction in bcc structure.

3. Atomic packing fraction for Face Centred Cubic structure (FCC)

Fig.1.13:face centered cubic.

The number of atoms is given by:

6 8
 Z   3 1  4  Z 4
2 8

Each face the atoms touches each other. Along the diagonal of face we have D=4r ,
where r is the radius of an atom. By geometry, the length of the face diagonal is D= a√2.

the fore from these two relation, we found the radius of atom as:

4
Z     r3
a 2 3 4   a3  2  2  2
r  there fore f   4     0, 74
4 a3 3 4  4  4  a3 6

 f  74 % which is the packing fraction of the FCC structure.

11
1.2. FORMS OF BONDING AND ENERGY BAND MODEL IN SOLIDS.
The next question we will be asking ourselves is what holds atoms together to make a
crystal and how to explain the differences in conductivity.

1.2.1 Forms of bonding.

Crystal may classified into about four groups in terms of the type of chemical bonding
forces keeping their atoms, ions or molecules together. The distinct types of bonds that
provide the cohesive forces in crystal are: the ionic bond, the covalent bond, the metallic
bond, the Van del Waals bond. The properties of a crystalline solid depend strongly upon
the nature of the chemical involved.

a. The ionic bond.


The ionic bond is formed when electrons are transferred from one atom to another,
forming charged ions which are attracted to each other by electrostatic Forces.
Ionic bonding involves transfer of an electron from one atom (which becomes a positively
charged = cation) to another (which becomes a negatively charged = anion). The two
ions attract strongly to form a crystal lattice.

Figure 1.16: The ionic bond.


It is observed in crystal structure made by positive and negative ions placed in alternate
locations. A typical example of ionic crystal is the sodium chloride and mainly elements
of group I and VII.

Fig.1.17: An example of ionic crystal

12
The arrangement of positive and negative ions in 2 dimension plane of sodium chloride.
The principal contribution to the cohesive energy in ionic crystal is the electrostatic
potential energy resulting from the coulomb attraction between the ions. For NaCl
crystal, the potential energy is given by the following expression:

e2
U 
4 0 r

where α is the Madelung constant of the crystal, is equal to -1.748 and r is the average
of interatomic distances. Constant α is named after Erwin Madelung, a German
physicist.

b. The covalent bond.


In a covalent crystal, the atoms are bound together by directional bonds resulting from
the sharing of electrons.

Figure 1.18: The covalent bond.

Diamond with four binding electrons is an example of covalent crystal, but elements of
group IV such as silicon and germanium have covalent bonds as well.

Fig.1.19: An example of covalent bonding in crystal

Covalent bonds are very strong compared to ionic bonds. All covalent crystals are hard,
insoluble in ordinary liquid and have a high melting point.

13
c. The metallic bond
Metallic bond, force that holds atoms together in a metallic substance. Such a solid
consists of closely packed atoms. In most cases, the outermost electron shell of each of
the metal atoms overlaps with a large number of neighbouring atoms. As a consequence,
the valence electrons continually move from one atom to another and are not associated
with any specific pair of atoms. It is a force of attraction between valence electrons and
the metal ions. It is the sharing of many detached electrons between many positive ions,
where the electrons act as a "glue" giving the substance a definite structure. It is unlike
covalent or ionic bonding.
In metallic bonding electrons freely move around a metal lattice (metal atoms close
together) to form a bond between all of the metals. The metallic bond arises when all of
the atoms share all of the valence electrons. These valence electrons are common to the
entire aggregate. The crystal is held together by the electrostatic attraction between the
negative electron and the positive metallic ions.

Fig.1.20: metallic bond.


The best example of metallic crystal is sodium.

Fig.1.21: An example of metallic crystal: The sodium crystal.

A common characteristic of metallic elements is they contain only one to three


electrons in the outer shell. When an element has only one, two or three valence
electrons (i.e. electrons in the outer shell), the bond between these electrons and the
nucleus is relatively weak.

So, for example, when aluminium atoms are grouped together in a block of metal,
the outer electrons leave individual atoms to become part of common “electron
cloud.” In this arrangement, the valence electrons have considerable mobility and
are able to conduct heat and electricity easily.

14
d. The Van Der Waals bond.
The Van Der Waals Force to put it simply is the sum of the negative and positive charges
between different molecules. Just like the net charges of electrons shells makes bonds
between atoms, The Van Der Waals force does the same for molecules. Van der Waals
forces are very weak compared to ionic or covalent forces.

The van Der Waals' forces is different from the forces that make up the molecule. For
example, a water molecule is made up of hydrogen and oxygen, which are bonded
together by the sharing of electrons. These electrostatic forces that keep a molecule
intact are existent in covalent and ionic bonding but they are NOT van der Waals'
forces.

The van Der Waals' forces are the forces that exist between the millions of separate water
molecules, and not between the hydrogen and oxygen atoms in the case of water.

Figure1.22: Van Der Waals bond.

One example is this as you know the solar system has planets, they rotate around the
sun and keep distance each another. There are two forces called centrifuge and
centripetal, those forces are responsible for the planets to stay in a stable way to rotate
without go out of their orbits, is the same with the Van Der wall forces but of course in
a molecular level. The Van Der Waals bond is mainly observed in inert gases where
neutral atoms with closed electron shells are bound together by the Van Der Waals
forces.

1.2.2 ENERGY BAND THEORY IN SOLIDS.

a. Introduction

15
As previously discussed, solids are composed by atoms, an atom consists of electrons
orbiting around a nucleus. However, the electrons cannot choose any orbit they wish.
They are restricted to orbits with only certain energies. Electrons can jump from one
energy level to another, but they can never have orbits with energies other than the
allowed energy levels.
According to quantum theory, electrons in an atom revolve around the nucleus in
stationary orbits with discrete energy levels. Electrons are not allowed to gradually loose
or gain energy. The transitions between energy levels are only allowed when electron
absorb or emit a quantum of energy.

Furthermore Pauli’s exclusion principle set a finite number of electrons in each orbit.
Therefore the energy levels in atoms combined with the number of electrons in each
energy level are specified. The maximum number of electrons a shell can have is equal
to 2n 2 . Hence for a K-shell we have:

 n=1 it can have a maximum of  2  12  2 .

 n=2 it can have a maximum of  2  2 2  8 .

 n=3 it can have a maximum of  2  32  18 .

 n=4 it can have a maximum of  2  4 2  32 etc.

The filling up of electrons in energy levels is in such a way that the system tends to be in
a minimum energy state and therefore electron will start filling the lowest energy level.
Once the allowed slots are filled in this level the next electron will go to the second until
the available electrons are used up. Therefore the electronic structure in an atom
comprises the filled energy levels where all slots are completely occupied, others which
are partially filled levels where there are some electrons but all slots are not filled and
finally the empty levels which are available for electrons but no electron is present.
Those different energy levels are separated by forbidden energy gaps as indicated on the
figure below.

Fig. 1.23 a) Energy levels in isolated atoms. b) Energy levels in crystal solids.

When atoms are brought together to form a solid crystal, the electrons in the upper
levels of adjacent atoms interact to bind the atoms together. These orbitals merge into
energy bands and electrons in those bands are not tied to an atom but they are free to

16
move around the crystal if the Pauli Principle allows. In addition to that those shared
electrons can have continuous values of energy within the energy band.

As the crystal contains many hundreds of millions of atoms, many energy levels are
associated with each nucleus and the appropriate energy level diagram for the entire
crystal has a band of energy levels.

The highest filled energy band in the crystal is called valence band. Conversely, the next
allowed energy band is empty of electrons because it corresponds to the unoccupied
higher levels in the isolated atom. It is called the conduction band. The energy region
between the valence and conduction band is called forbidden energy gap since no
electrons with such energies exist in the crystal. At absolute zero temperature, all
electrons fill up the lowest energy levels, and the highest filled level is called the Fermi
level.

While the general features of the energy band model for any solid crystal are as
described, many important properties depend upon the specific atoms and crystal
structures. In particular, the differences between metals, semi-conductors and
insulators are reflected in their energy band models.

Fig.1.24: Simplified diagram of energy band in solids where holes in the valence band and electrons in
the conduction band are highlighted.

b. Conductors, insulators and semi conductors.


 Conductors: Starting with metals, the atomic and crystal structure of metals are
such that the valence and conduction bands overlap, therefore show no
forbidden energy gap. At normal temperature, some electrons from the valence
energy band are excited to levels in the conduction energy band and therefore
metals are good conductors.

Most of the conductors used in electronics are metals like copper, aluminium and
steel. Conductors are materials that obey Ohm's law and have very low resistance.
They can therefore carry electric currents from place to place without dissipating
a lot of power.

 Insulators: An insulating crystal has a wide forbidden energy gap. Obviously


the upper energy band cannot contribute to electric conductivity since no
electrons are present to act as carriers.

In terms of energy bands, it means that insulators have:

- full valance band

17
- an empty conduction band, and

- a large energy gap (of several eV) between them.

Examples of Insulators: Glass, most polymers (plastics), rubber and wood are all
examples of insulators. These are materials which will refuse to carry an electric
current.

 Semiconductor The energy band model of a semiconductor is similar to that of


an insulator except that the forbidden energy gap is comparatively narrow.
Thermal energy at room temperature is sufficient to promote few electrons from
the valence band to the conduction band across the forbidden energy gap. The
promoted electrons can conduct electricity and the corresponding electron
vacancies or holes in the valence band make it possible for electron in this band
to contribute to conductivity as well.

In terms of energy bands, semi-conductors can be defined as those materials


which at room temperature have:

- partially- filled conduction band

- partially- filled valence band and

- a very narrow energy gap( of order of 1eV) between them.

at 0°K, there are no electrons in conduction band of semiconductors and their


valence is completely filled.

Example of semiconductors are: All the transistors, diodes, integrated circuits,


etc. used in modern electronics are built using a range of semiconductors. The
semiconductor conducts a little bit. A semiconductor will carry electric current,
but not as easily as a normal conductor.

18
Figure25: Insulator, semiconductor and conductor.

At a very low temperature semi-conductors become insulator. The width of the forbidden
energy gap is of the order of 5 eV.

Table 1 gives the examples of energy gaps in the most encountered elements or
compounds.

Element or compound Energy band gap (eV)

silicon 1.1

germanium 0.7

Gallium arsenide (GaAs) 1.34

Indium antimonide (InSb) 0.18

Cadmium sulfide (CdS) 2.45

Zinc Oxide (ZnO) 3.3

Table 2: Energy gaps of the most encountered elements or compounds.

Materials with a wide forbidden energy gaps are desirable for semi-conductor devices.
The number of electrons promoted to the conduction band at high temperature is small
and the change in device characteristics with temperature is less severe. Silicon is the
most widely used than germanium even though the latter is easier to prepare and less
expensive.

c. Electrons and holes


The net current resulting from electrons in a filled valence band is zero. Then the current
density j is written as:

j  nev  0, (1.1)

19
where n is the density of electrons, e the electronic charge and v the average velocity of
the electrons in the band. The expression of the current density can be rearranged as
follow

1 n n
j  ne  i i vi  0,
n i 1
v  e
(1.2)

Electrons in the valence band of a semi-conductor at room temperature can conduct


current because of the few vacant levels left behind by electrons excited to the
conduction band. If we focus our attention upon the jth electron in above equation, we
can get

n n
j  e vi  e vi  ev j  0
(1.3)
i 1 i 1
i j

This expression can be rearranging into the following equation:

e vi  ev j (1.4)
i 1
i j

here e v
i 1
i is the current resulting from all the electrons in the valence band except
i j

for the jth electron,  ev j is the current density due to one electron but of opposite
charge. This vacant level in the valence energy band is called hole.

The holes in the valence band can be treated as positively charged carriers fully
analogous to the negatively charged electron in the conduction energy band.

The conductivity of a semiconductor crystal containing both electrons in the conduction


band and holes in the valence band is given by:

  nee  peh (1.5)

where e and h are the respective mobilities of electrons and holes.

20
d. Intrinsic and extrinsic semi-conductors.
 Intrinsic semi-conductors.
The energy band model discussed above refers to a perfect semiconductor crystal
structure which contains no chemical impurities and in which no atoms are displaced
from their proper sites. An intrinsic semiconductor is ideally a perfect crystal.

The properties of such solid crystal are therefore characteristics of an ideal structure.
Such crystals are called intrinsic semi-conductors. The number of electrons in the
conduction band equals the number of holes in the valence band since both are the
result of electron transitions across the forbidden energy gap.

It can also be noticed that the number of charge carriers or number of transitions
between the valence and conduction energy bands increases with increased
temperature. Thus the conductivity of intrinsic semiconductors is a function dependent
of the temperature.

Example.

Pure silicon is an example of an intrinsic semiconductor. Silicon is a group IV element,


and has 4 valence electrons per atom. In pure silicon the valence band is completely
filled at absolute zero.

Fig.1.26. An intrinsic semiconductor.

When an electrical field is applied to an intrinsic semi-conductor at a temperature


greater than absolute temperature, the conduction electrons move to the anode while
holes in the valence band move to the cathode. Hence the current in intrinsic semi-
conductors consists of movement of electrons and holes in opposite direction in the
conduction and valence band respectively.

 Extrinsic semi-conductors.

The electrical properties of a semiconductor are drastically altered when foreign atoms or
impurities are incorporated into the crystal. The solid crystal is then called extrinsic
semiconductor since the properties now depends strongly upon the impurities.
Extrinsic semiconductors are actually intrinsic semiconductors to which suitable
impurities or doping agents have been added in extremely small amount.

21
Taking an example of a single crystal of silicon in which each atom has four valence
electrons and suppose now a pentavalent atom, such as phosphorus, arsenic or
antimony is substituted to a silicon atom in the crystal. Four of the impurities atom’s
electrons play the same role as the four valence electron of the replaced silicon atom and
become part of the valence band. The fifth valence electron of the impurity is easily
detached by thermal energy and move freely in the conduction band.

 Phosphorus, arsenic, bismuth and antimony impurity atoms in silicon crystal donate
electron to the conduction band and are called donor impurities. The resulting
semiconductor is termed n-type extrinsic semiconductor (Fig. 1.27).

 If the doping agents are trivalent atoms such gallium, indium, aluminum, boron etc.
which have three valence electrons are called acceptor atoms. The resulting
semiconductor is p-type semiconductor (Fig1.28).

e. Majority and minority charge carriers


At absolute zero temperature, intrinsic semi-conductors have no free charge carriers
available but as the temperature rises to room temperature, some of the covalent bonds are
broken by thermal energy and as a result, electron-hole pairs are produced. Those are also
known as thermally generated charge carriers.

22
An intrinsic or pure semi-conductor can be converted to a p-type semi-conductor by
addition of acceptor impurities which add a large number of holes in it. Similarly an
intrinsic semiconductor can be converted to a n-type semiconductor by addition of donor
impurities which add a large number of conduction electrons in it.

Hence each extrinsic p-type semi-conductor has large number of holes or added impurities
namely majority carriers and small number of electron or minority carriers. On the converse
N-type semiconductor has electrons as majority carriers while holes are minority carriers.

f. Drift current in intrinsic and extrinsic semi-conductor


If an electric field is applied to a conductor, it will cause the electrons to drift toward the
positive terminal of the applied voltage even if it doesn’t stop collisions and random motion of

electrons. The average drift velocity vd will be proportional to E

vd  e E (1.6)

where  e is mobility of electron. In general, the electrical mobility is the ability of charged
particles (such as electrons or protons) to move through a medium in response to an electric
field that is pulling them. The SI unit of mobility is m2/(V·s).

The equation(1.1) the current density for valence band, was written as:

j  nev  0,

Thus for a conductor with a cross section (A), the drift current will be given by

I  j A

The above relations can be combined to obtain the drift current as:

I  enAv  neAe E
(1.7)

But as

V
E (1.8)
l

where l is the length of the conductor and V the electric potential across the conductor.

V
I  neAe (1.9)
l

According to Ohm's law, in the above equation we can write the net resistance R as follow:

V l (1.10)
R 
I Ane e

At the same time, this resistance can be expressed in terms of the length of the conductor l,
the resistivity of the conductor ρ and the cross section area of the conductor A as follow.

23
l
R (1.11)
A

Comparing the two previous equations we deduce that

1
 (1.12)
ne e

The resistivity is expressed in Ohm-meter (  m ) and therefore the conductivity σ which


is the inverse of the resistivity is given by

  nee (1.13)

The conductivity of conductors is expressed Siemens/meter.

In extrinsic semiconductors, the current is due to the movement of electrons and holes
in opposite direction. The net current is therefore the algebraic sum of both
contributions;

I  I e  I h  ne e A  ep h A
(1.14)
I  ne e   h A  nee  h A

In the above equation we can find the resistance as:

V l 1
R  
I A ene  h 
(1.15)
1

nee  h 

A low resistivity indicates a material that readily allows the movement of electric charge

And the conductivity is given by:

  ne( e   h ) (1.16)

Electrical conductivity or specific conductance is the reciprocal of electrical resistivity,


and measures a material's ability to conduct an electric current. It is commonly
represented by the Greek letter σ (sigma). Its SI unit is Siemens per metre (S/m) or
reciprocal second (s−1).

24
CHAPTER 2: SEMICONDUCTOR DEVICES.

Introduction

Having discussed the properties of semiconductor and particularly electronic properties of


these materials, it becomes easier to understand the basic principle of devices fabricated
with these materials. In this chapter some semiconductor devices of great importance in our
daily life will be discussed in the following steps starting from the simplest to the more
elaborated: p-n junction diode, bipolar junction transistor and amplifiers

2.1. PN-Junction diode

In this section we will mainly describe a p-n junction and the working principle of ordinary
and zener diode. Different applications of p-n junction diode and Zener diode such a
rectifiers, filters and voltage regulators are also discussed in this section. A p-n junction is
formed by bringing together a p-type and a n-type semiconductors in a very close contact.
The junction is the contact surface between the two pieces.

Fig.2.1: A p-n junction.

Since in the n-type has a high concentration of electrons and the p-type has a high
concentration of holes, each type of carriers will diffuse from the high concentration towards
the region of lower concentration across the junction.

Since both carriers are charged, the diffusion will not continue until both carriers are the
same on both side of the junction. When electrons and holes move to either sides of the
junctions, they will leave behind exposed charges on doping atoms sites which are immobile
in the crystal.

25
Fig.2.2: The depletion layer around a p-n junction.

An electric field will therefore develop between the fixed positive and negative ion cores. This
region is called the depletion region since quickly the electric field sweeps free carriers out
and therefore depleting the region from free carriers.

The formation of the depletion layer is due to the difference in concentration which
establishes a density gradient across the junction resulting in diffusion of majority carriers
from both sides of the junction. The holes diffuse from the p-type region towards the n-type
region while electrons diffuse from n-type to p-type region. They will terminate their
existence by recombination resulting in the depletion layer. The width of the depletion layer
depends on the concentration of majority carriers and the diffusion coefficient of these
carriers.

A potential barrier or built-in potential associated with the developed electric field will be
formed at the junction. Typically for Si at room temperature, the potential barrier is in the
range of 0.7eV and the width of the depletion layer is in the range of 0.1µm to1µm

2.2. Forward and reverse biasing of a P-N junction.

The basic difference between a forward bias and reverse bias is in the direction of applying
external voltage. The direction of external voltage applied in reverse bias is opposite to that of
external voltage applied in forward bias.
We have learned that if we apply an external voltage higher than the barrier potential of p-n
junction diode, it will start conducting, which means it will start passing current through it.
So how we are going to study the behaviour of PN- junction diode under forward and reverse
biased condition
 Forward bias: A P-N Junction is said to be biased in the forward direction when
the positive battery terminal is connected to p-region of the semi-conductor and the
negative battery terminal to the n-region.

26
Fig 2.3 : Forward bias.

What really happens inside the diode during the forward bias ? We know a diode has a
depletion region with a fixed barrier potential. This depletion region has a predefined
width, say W. This width will vary for a Silicon diode and a Germanium diode. The width
highly depends on the type of semiconductor used to make P-N junction, the level of doping
etc.

When we apply voltage to the terminals of diode, the width of depletion region slowly starts
decreasing. The reason for this is, in forward bias we apply voltage in a direction opposite to
that of barrier potential. We know the P-side of diode is connected to positive terminal and
N-side of diode is connected to negative terminal of battery.

So the electrons in n-side gets pushed towards the junction (by force of repulsion) and the
holes in p-side gets pushed towards the junction. As the applied voltage increases from 0
volts to 0.7 volts, the depletion region width reduces from ‘W’ to zero. This means depletion
region vanishes at 0.7 volts of applied voltage.

This results in increased diffusion of electrons from N-side to P-side region and the increased
diffusion of holes from p-side to N-side region. In other words, “minority carrier” injection
happens on both N-side (in a normal diode (without bias) electrons are a minority on P-side)
and N-side (holes are a minority on N-side) of the diode.

In this case, majority carriers are repelled by the positive and negative battery terminal with
the result that both carriers are driven toward the junction and cross it. There is a large
current flow through the semi-conductor and obviously the crystal offers low resistance in
this forward direction. Note that only free electrons move in the external circuit. Thus, the
forward bias direction reduces the thickness of the depletion layer around the p-n junction.

 Reverse bias: When a battery is connected to junction diode with p-region connected to
negative pole and N- section connected to the positive pole, the P-N junction is said to be
reverse biased. In this set up the holes are attracted by the negative battery terminal and
electron by the positive terminal and the majority carriers do not cross the junction. There is
no electron hole combinations except for minority carriers as electron and holes move away
from the junction. The width of depletion layer increases the potential barriers and the
junction offers high resistance.

27
Fig 2.4: Reverse bias of p-n junction.

2.3.Current- voltage characteristics of a P-N junction.


These are graphs drawn between the bias voltage and the circuit current of a junction diode.

Fig. 2.5: The I-V characteristic of a p-n junction.

 When the forward bias voltage is gradually increased in steps above the barrier voltage, the
current increases rapidly approximately exponentially.

 Under reverse bias, although there is practically no current due to majority carriers, yet
there is a small amount of current due to the flow of minority carriers across the junction
and this current is called reverse saturation current and it is extremely temperature
dependent.

If the reverse bias voltage applied to a P-N junction is increased, a point is reached when the
junction breakdown and the reverse current rises sharply to a value limited only by the
external resistance connected in series with the junction.

28
This critical value of the voltage is known as breakdown voltage. The junction offers almost
zero resistance at this point. A very little increase in voltage increases the reverse current to
relatively high values. There are 3 mechanisms leading to the junction breakdown:

i. Thermal instability.
When the voltage is increased further in the reverse direction, the thermal energy increases.
It may reach the point where it can break covalent bonds and thus releases more charge
carriers. Hence the reverse current increases. This current increases again the heat and so
on the current increases more. This may lead to thermal breakdown or thermal run away.

ii. Zener breakdown or Zener effect.


It occurs in junction which, being heavily doped, have narrow depletion layers. The
breakdown voltage sets up a very strong electric field across the narrow depletion layer.
This electric field is strong enough to break or rupture the covalent bonds in the
semiconductor and hereby generating election-hole pairs. The junction therefore has a very
low resistance in the breakdown region.

iii. Avalanche breakdown.


It occurs in junctions which, being lightly doped, have wider depletion layers. The electric
field is not strong enough to produce Zener breakdown instead minority carriers will be
accelerated by this electric field and collide with the semiconductor atoms in the depletion
layer. Upon collision with valence electron, covalent bonds are broken and electron-holes
are generated. The newly generated charge carriers are also accelerated by the electric field
resulting in more collisions and hence further production of charge carriers. There is an
avalanche or flood of charge carriers across the junction which shows a very low resistance.

2.4. The PN- junction diode.

A diode is the simplest electronics device with two-terminal electronic component, one
component is p-type and other n-type. The diode will transfer current in one direction (
resistance in this direction is small) but it will have high resistance in other side and will not
permit current to follow in this side.

This is the simplest semiconductor device. It is a two terminals device consisting of a p-n
junction formed either in germanium or silicon crystals. The figure below is the symbol of
diode.

Fig.2.6: Ordinary crystal diode symbols.

29
A PN- junction diode is a one way device offering low resistance in forward biased and
behaving almost as an insulator in reverse biased. As it can be discussed later this property
allows the diodes to be used as rectifiers for converting alternating current into direct
current. The static voltage- current characteristic for a P-N junction diode is obtained as
follow:

Fig.2.7: The static current voltage characteristics for ordinary diode

 When the diode is forward biased and the applied voltage is increased from zero, hardly any
current flows through the device in the beginning. The external voltage is being opposed by
the internal barrier voltage. As soon as the barrier potential is neutralised, the current
through the diode increases rapidly with increasing applied voltage.

 When the diode is reverse biased, majority carriers are blocked and only small current due to
minority carriers flows through the diode. As the voltage is increased from zero in the reverse
direction, the reverse current very quickly reaches a maximum or saturation value I 0 known
as leakage current. When the reverse voltage exceeds a certain value called break down
voltage Vz , the leakage current suddenly and sharply increases. The curve indicates quite a
zero resistance at this point.

In the forward bias region, the power of the diode is given by:

P  I DVD (2.1)

This power is dissipated in the diode as heat. With increased current or voltage, the diode
will reach a point when the heat generated can no longer be cooled down by air and its
temperature will rise dramatically and the diode will burn out. Under reverse bias the regular
diode is usually destroyed when operated in the reverse breakdown region

30
CHAPTER III. DIODES AS RECTIFIERS OF CURRENT.
Most of electronic devices and circuits requires a dc source for their operation. Dry cells and
batteries are one of the form of dc source. They have the advantages of being portable and
ripple free. However , their voltage are low, they need frequent replacement and are
expensive as compared to conventional dc power supplies. Since the most convenient and
economical source of power is the domestics ac supply, it is advantageous to convert this
alternating voltage (usually, 220V rms) to dc voltage( usually smaller in value). This process
of converting ac into dc voltage is called rectification and is accomplished with the help of
a. Rectifier .
b. Filter, and
c. Voltage regulator circuits.
These elements put together constitute dc power supply.
In practice rectifier are associated with transformers which allow stepping up or stepping
down the voltage.
3.1. Transformer.
When electric power is transmitted over great distances, it is economical to use a high
voltage and low current to minimise the power ( IR2) loss in the transmission lines. at the
receiving end of such lines, the consumer requires power at low voltage (for safety and for
efficiency in design). Therefore a device is required that can change the alternative voltage
and current without causing appreciable changes in power delivered. That device is called an
AC transformer.
A transformer is a device used to increase or decrease the AC voltage in a circuit. A typical
device consists of two coils of wire, a primary and a secondary, wound around an iron core,
as illustrated in figure below.

Iron core

Figure 3.1: A transformer.

 The primary coil, with N1 turns, is connected to alternating voltage source


V1(t). The coil N1 is called primary winding (or the primary).
 The secondary coil has N2 turns and is connected to a “load resistance” R2. The
coil N2 is called secondary winding (or the secondary).
The way transformers operate is based on the principle that an alternating current in
the primary coil will induce an alternating emf on the secondary coil due to their
mutual inductance.

In the primary circuit, neglecting the small resistance in the coil, Faraday’s law of
induction implies that:

31
d B
V1   N1 (3.1)
dt

where ΦB is the magnetic flux through one turn of the primary coil. The iron core,
which extends from the primary to the secondary coils, serves to increase the magnetic
field produced by the current in the primary coil.
Ensure that nearly all the magnetic flux through the primary coil also passes through
each turn of the secondary coil.

 Thus, the voltage (or induced emf) across the secondary coil is given by:

d B
V2   N 2 (3.2)
dt

In addition, no magnetic flux leaks out from the iron core, and the flux Φ B through
each turn is the same in both the primary and the secondary coils. Combining the two
expressions, we are lead to the transformer equation:

V2 V V2 N 2
 1 or  (3.3)
N 2 N1 V1 N1

By knowing that the power in primary coil is equal to the power in secondary, this lead
to the equation

V2
I 1V1  I 2V2  I1  I2 (3.4)
V1

Putting equation 5.3 in 5.4. the transformation of currents in the two coils may be
obtained as:

N 
I 1   2  I 2 (3.5)
 N1 
Thus, we see that the ratio of the output voltage to the input voltage is determined by
N2
the turn ratio . If N 2  N 1 then V 2  V1 , which means that the output voltage in
N1
the second coil is greater than the input voltage in the primary coil. A transformer with
N 2  N 1 is called a step-up transformer. On the other hand, if N 2  N 1 , then V 2  V1
and the output voltage is smaller than the input. A transformer with N 2  N 1 is called
a step-down transformer.

we can now understand why transformers are useful for transmitting power over long
distances. Because the generator voltage is stepped up, the current in the transmission
line is reduced , and hence power losses are reduced. in practice, the voltage is
stepped up to around 230 000V at the generating station, stepped down to around 20
000 V at a distribution station.

There is a practical upper limit to the voltages that can be used in transmission lines.
excessive voltages could ionise the air surrounding the transmission lines, we could

32
result in a conducting path to ground or to other objects in the vicinity. For this
reason , a long string of insulators is used to keep high voltage wires away from their
supporting metal towers. Others insulators are used to maintain separation between
wires.

Figure 3.2: Generating station transformer.

3.2. RECTIFIER CIRCUITS.

A rectifier circuit is a device which convert alternating current (ac) into direct current
(dc). The p-n function diode is used in rectifier circuit because it allows current to flow
easily in one direction only. There are two types of rectifiers namely:

i. Half wave rectifiers which rectify a half wave of the input signal and

ii. Full wave rectifiers which transform the whole wave signal.

A perfect diode in the forward connection acts like a closed switch but in reverse
connection it acts like an open switch. However, in practice, there is no perfect or ideal
diode; hence we must take into account the potential barrier VB when the diode is
forward biased and the effect of the forward dynamic resistance of the diode. In the
reverse direction, we consider a high reverse bias resistance and a low saturation
current for voltages below the breakdown voltage. We recall that the reverse resistance
of a diode is of the order of mega ohms.

3.2.1. Half wave rectifier.

When a single rectifier unit( or diode) is placed in series with the load across an ac
supply, it converts alternating voltage into uni-directional pulsating voltage, using one
half cycles of the applied voltage, the other half cycles being suppressed because it
conducts only in one direction.

33
Fig.3.4 a) Half wave rectifier b) Output signal from a sinusoidal input signal
During the half cycle of the original signal from time 0 to the time x, the diode is forward
biased and conductors a current I through the circuit. During the next half cycle from x to
2x, the diode is reversing biased and doesn’t conduct any current. Hence the output signal
consists of half cycles separated by zero signals. This is called half wave rectification.

The effect of the barrier potential on the output signal VRL is to reduce it by Vb. (Vb is the
beak down voltage of diode).

VRL  VP  Vb
(3.6)

Where Vp is the input peak voltage and Vb=0.7V for silicon diodes.

In practice rectifier are associated with transformers which allow stepping up or stepping
down the voltage. Its job is either to step up or ( mostly) step down the ac supply voltage to
suit the requirement of the solid-state electronics devices and circuits fed by the dc power
supply. It also provides isolation from supply line-an important safety consideration.

Fig.3.5: A half wave rectifier with a transformer.

A half wave rectifier is less efficient and needs more filtering , therefore another kind of
rectifier is developed to make more filtering and is known as full-wave rectifier.

3.2.2. Full wave rectifier.


There are two types of full wave rectifiers: Centre tapped full wave rectifier and Full wave
bridge rectifier.

34
A. Centre tapped full wave rectifier.

It is made up of two diodes which are appositely connected and may be connected to a
transformer.

In such a rectifier, the ac input is applied through a transformer, the anodes of the two
diodes D1 and D2 (having similar characteristics) are connected to the opposite ends of the
centre tapped secondary winding and two cathodes are connected to each other and are
connected also through the load resistance RL and back to the centre of the transformer, as
shown in figure below.

Fig.3.6: The working principle of a full wave rectifier with a transformer.

During the positive half-cycle from 0 to  the diode D1 is forward biased and drive a current
through the load resistance RL. During the next half cycle from  to 2  the other diode D2 is
forward biased and allows current through the load resistance Rl.

B. Full wave bridge rectifier. It is made up of four diodes.

Fig. 3.7: The working principle of a full wave bridge rectifier.

During the first cycle from t0 (say 0 sec) to t1 (say 0.01 sec), the diodes D1 and D3 are
forward biased and allow the current to flow through the load resistance RL in the direction
shown by the arrow in Fig. 3.7.b. The diodes D2 and D4 do not conduct because they are

35
reverse biased act like open switches. During the next half-cycle from t1 to t2 (say 0.02 sec)
the diodes D2 and D4 are on and a positive voltage appears on the load resistance RL. D1
and D3 are reverse biased and hence act as an open switch. The resulting voltage from the
AC input waveform is shown on the figure below.

Fig.3.8: a) Input AC waveform signal b) The output positive voltage.

Note that since there are two diodes in series, the effect of potential barrier on the output
signal will be doubled.

The reverse bias is half of that in the centre tapped rectifier because of the lower peak
inverse voltage in the bridge rectifier. Hence the bridge rectifier is preferred to the centre-
tapped full wave rectifier.

3.3. FILTERS AND RIPPLE FACTORS.

Introduction.

A power supply must provide ripple free source of power from an A.C. line. But the output of
a rectifier circuit contains ripple components in addition to a D.C. term. It is necessary
to include a filter between the rectifier and the loads in order to eliminate these ripple
components. The filter is a device which used to try to remove the ripples and produce as
smooth signal as possible.

Fig.3.9: Illustration of the working principle of a filter.

Ripple components are high frequency A.C. Signals in the D.C output of the rectifier. These
are not desirable, so they must be filtered. So filter circuits are used. In practice there are
four types of filters:

 Shunt capacitor filter or RC -filter.

 Series inductor filter or L-filter.

 Chock input or LC filter.

 Pi(π) filter or CLC filter.

36
The RC filter uses a capacitor; the inductor filter uses a single inductor in its filter’s circuit
while LC filter uses both the inductor and capacitor in its circuit. The π -filter is somehow a
combination of LC and RC filter circuits.

3.3.1. Shunt capacitor filter (C-filter)


This type of filter consists of large value of capacitor connected across the load resistor RL as
shown in figure below.

Fig.3.10: Shunt capacitor filter.

For the shunt capacitor filter, a capacitor will acts as an open circuit for direct current (DC).

 During the half cycle from t0 to t1 , the diode is forward biased and current flows into
the capacitor C. This current charges fully the capacitor.

 And during the next half cycle from t1 to t2 , when the input from the diode is zero, the
capacitor discharges and drives a current through the load R1 . Hence the capacitor
tries to reduce the ripples as it is shown on the figure below.

Fig. 3.11: Working principle of a shunt capacitor filter (A-C).

37
Fig.3.12. The final output signal

For a full wave rectifier, the smoothing effect is even better because in this case the
discharge time of the capacitor is only half of that of a half wave rectifier.

The ripple factor for the shunt capacitor filter is given by the formula:

1
r
4 3 fCRL (3.7)

where f is the frequency of the signal, C is a capacitance and is expressed in µF and RL in


ohms.

These types of filters are used in circuits with small load current like transistor radio
receivers, calculators, etc.

Advantages

 Low cost.

 Small size and weight.

 Good characteristics.

 Can be connected for both HW and FW rectifiers.

 Improved d.c. output.

Disadvantages

 Capacitor draws more current.

3.3.2. Series inductor filter (L-filter).

The working of series inductor filter depends on the inherent property of the inductor to
oppose any variation in current intend to take place.

Figure below shows a series inductor filter connected at the output of a Full Wave Rectifier
(FWR). Here the reactance of the inductor is more for ac components and it offers more
opposition to them. At the same time it provides no impedance for d.c. component. Therefore
the inductor blocks a.c. components in the output of the rectifier and allows only d.c.
component to flow through RL.

38
Fig.3.13:a. Working principle of a series inductor filter b. The output current.

When a changing voltage is applied to an inductor, the induced current will opposes the
change in the current. This means that when the voltage falls, the induced current raises in
the region of the output where there was no current.

The appreciation on how a good a filter is can be determined by the estimation the ripple
factor as the output current is not completely smooth and also show some ripples
superimposed on the dc signal but with small amplitude ( see figure below).

Fig.3.14: Ripples superimposed on the dc signal.

The ripple factor is defined as follows:

riple voltage Vr
riple factor   r
Dc voltage Vdc (3.8)

where Vr is the ripple peak to peak voltage , Vdc is the resulting dc voltage.

For the series inductor filter the ripple factor is given by the following expression.

RL
r
3 2L (3.9)

where   2 f is the frequency of the signal. The ripple factor varies with the load
resistance.

Advantages.

 Sudden changes in current is smoothen out.

 Improved filtering action at high load currents.

39
Disadvantages

 Reduced output voltage due to the drop.

 Improved filtering action at high load currents across the inductor.

 Bulky and large in size.

The difference between shunt capacitor and series inductor filter:

Parameter Shunt Capacitor Series Inductor

Place of filter Across the load In series with load

Useful in Reducing ripple in load Reducing ripple in load


voltage current

Lowest ripple in load No load or light loads Heavy loads


voltage at

Suitable for Light load application Heavy load application

Surge current Very high and must be Low and need not be
controlled control

Ripple factor R = 1/ 4√3 FCR R = R/3√2 WL

Size of filter Small and compact Bulky

3.3.3. LC Filter.

It is a combination of inductor and capacitor filter. Here an inductor is connected in series


and a capacitor is connected in parallel to the load as shown in fig above.
As discussed earlier, a series inductor filter will reduce the ripple, when increasing the load
current. But in case of a capacitor filter it is reverse that when increasing current the ripple
also increases. So a combination of these two filters would make ripple independent of load
current.

Fig.3.15: Typical LC filter.

40
The capacitor offers a very low reactance to the ac ripple current and hence shunts most of it
to the ground. However it appears as open to the dc current and forces it to pass through the
inductor and the load resistance.

This type of filter combines the advantages of series inductor and shunt capacitor filters.
The output of this filter has very little ripples and good regulation.

Since the d.c. resistance of the inductor is very low it allows d.c. current to flow easily
through it. The capacitor appears open for d.c. and so all d.c. component passes through it
and through the load resistor RL.

The ripple factor for this type of filter is given by:

2
r
12 LC
2
(3.10)

By taking f=50Hz, This ripple factor will be equal to:

1.194
r
LC (3.11)

Advantages

 Reduced ripples at the output

 Action is independent of load current

Disadvantages

 Low output voltage

 Bulky and large in size

 Not suit to connect with HWR.

3.3.4. The π-filter (or CLC filter).


This type of filter uses two capacitors and an inductor. Hence this is combination of the
capacitor filter and LC filter. This kind of filter known as  -filter, produces a better signal
than any one of the previous filters but it is more costly.

Fig. 3.16: Typical π-filter

41
Here the first capacitor C1 offers a low reactance to a.c. component of rectifier output but
provide more reactance to d.c components. Therefore most of the a.c. components will
bypass through C1 and the d.c. component flows through chock L. The chock offers very
high reactance to the a.c. component. Thus it blocks a.c. components while pass the d.c. The
capacitor C2 bypasses any other a.c. component appears across the load and we get study
d.c. output as shown below.

The ripple factor of this filter is given by:

2
r
8 c1c2 LRl
3
(3.12)

or

2 X C1 X C 2
r
X L RL (3.13)

Where

1
XC 
C and X L  L (3.14)

Here C1 and C2 are in µF, L in henrys and RL in ohms.

Advantages

 More output voltage.

 Ripple less output voltage.

 Suitable to be used with both HWR and FWR.

Disadvantages

 Large in size and weight.

 High cost.

42
CHAPTER IV. ZENER DIODE.

Introduction.

In the previous section, we have seen that ordinary diodes are operating only in forward
direction or in reverse bias below the breakdown voltage. Beyond the breakdown voltage the
ordinary diode may get damaged. There is another type of diode which is designed to operate
even in reverse forward. This diode is called zener diode.

2.4.1. Zener diode.

However, the Zener Diode or “Breakdown Diode” as they are sometimes called, are basically
the same as the standard PN junction diode but are specially designed to be operating
Beyond the breakdown voltage. While ordinary diodes are used for rectification, the Zener
diodes are used for voltage regulation.

Fig 4.1: Circuit symbol of a Zener diode

A Zener diode is a diode which allows current to flow in the forward direction in the same
manner as an ideal diode, but will also permit it to flow in the reverse direction when the
voltage is above a certain value known as the breakdown voltage, "zener knee voltage" or
"zener voltage" or "avalanche point".

A Zener diode is a diode that is specially manufactured to operate under reserve breakdown
conditions. Under normal operation the Zener diode is reverse biased and the reversed-
voltage is greater than the breakdown voltage. The purpose of the Zener diode is to maintain
a constant output voltage. This output voltage as requires is equal to the breakdown voltage
Vz ( this properties of maintaining output voltage constant is called Voltage regulation).

The Zener diode is like a general-purpose signal diode consisting of a silicon PN junction.
When biased in the forward direction it behaves just like a normal signal diode passing the
rated current, but as soon as a reverse voltage applied across the Zener Diode exceeds the
rated voltage of the device, the diodes breakdown voltage is reached at which point a process
called Avalanche Breakdown occurs in the semiconductor depletion layer and a current
starts to flow through the diode to limit this increase in voltage.

The current now flowing through the zener diode increases dramatically to the maximum
circuit value (which is usually limited by a series resistor) and once achieved this reverse
saturation current remains fairly constant over a wide range of applied voltages. The voltage

43
point at which the voltage across the zener diode becomes stable is called the “zener voltage”
for zener diodes this voltage can range from less than one volt to hundreds of volts.

The point at which the zener voltage triggers the current to flow through the diode can be
very accurately controlled (to less than 1% tolerance) in the doping stage of the diodes
semiconductor construction giving the diode a specific zener breakdown voltage, (Vz ) for
example, 4.3V or 7.5V. This zener breakdown voltage on the I-V curve is almost a vertical
straight line.

2.4.2. Zener Diode I-V Characteristics.

Fig 4.2: I-V Characteristics

The Zener Diode is used in its “reverse bias” or reverse breakdown mode, i.e. the diodes
anode connects to the negative supply. From the I-V characteristics curve above, we can see
that the zener diode has a region in its reverse bias characteristics of almost a constant
negative voltage regardless of the value of the current flowing through the diode and remains
nearly constant even with large changes in current as long as the zener diodes current
remains between the breakdown current IZ(min) and the maximum current rating IZ(max).

This ability to control itself can be used to great effect to regulate or stabilise a voltage source
against supply or load variations. The fact that the voltage across the diode in the breakdown
region is almost constant turns out to be an important application of the zener diode as a
voltage regulator.

2.4.3. The Zener Diode Regulator.

Zener Diodes can be used to produce a stabilised voltage output with low ripple under
varying load current conditions. By passing a small current through the diode from a voltage

44
source, via a suitable current limiting resistor (RS), the zener diode will conduct sufficient
current to maintain a voltage drop of Vout.

Fig 4.3: Zener Diode Regulator.

The resistor, RS is connected in series with the zener diode to limit the current flow through
the diode with the voltage source, VS being connected across the combination. The stabilised
output voltage Vout is taken from across the zener diode. The zener diode is connected with
its cathode terminal connected to the positive rail of the DC supply so it is reverse biased
and will be operating in its breakdown condition. Resistor RS is selected so to limit the
maximum current flowing in the circuit.

The load is connected in parallel with the zener diode, so the voltage across RL is always the
same as the zener voltage, ( VR = VZ ). There is a minimum zener current for which the
stabilization of the voltage is effective and the zener current must stay above this value
operating under load within its breakdown region at all times. The upper limit of current is of
course dependent upon the power rating of the device. The supply voltage VS must be greater
than VZ. In practice its zener diode is said to have a determined power and from this power
we can find is maximum current. Therefore, according to Ohm's law:

P
P  V  I or I max  (1)
Vbreakvoltage

Example No1

We have a zener diode which has a power of 1 watt and its breakdown voltage of 0.7V.
therefore the maximum current it can have is given by:

P 1watt
P  V  I or I max    133mA .
V 0.7V

From the figure 4.3,

45
by using Kirchhoff's voltage and current rules, we have also the following equations:

Vin  VZ
Vin  RS I S  VZ  0 or RS  (2)
IS

By the right side of the above figure, the voltage across the road resistance is equal to voltage
across the zener diode, thus:

VZ
IL  (3)
RL

and from junction rules, the current across the source resistor is equal to:

IS  IZ  IL (4)

Example No2

A 5.0V stabilised power supply is required to be produced from a 12V DC power supply input
source. The maximum power rating PZ of the zener diode is 2W. Using the zener regulator
circuit above calculate.

a) The maximum current flowing through the zener diode.

b) The minimum value of the series resistor, RS

c) The load current IL if a load resistor of 1kΩ is connected across the zener diode.

d) The Zener current IZ at full load.

Solution

a) The maximum current flowing through the zener diode.

the maximum current flowing through the zener diode is equal to IS

I S  400 mA

46
It is reached when RL is very small and closer to zero.

b) The minimum value of the series resistor, RS

We get the minimum value of the series resistor for IS max.

c) The load current IL if a load resistor of 1kΩ is connected across the zener diode.

d) The zener current IZ at full load.

47
CHAPTER V. : BIPOLAR JUNCTION TRANSISTOR.

INTRODUCTION.

In 1947 J. Barden, W. Bratterin and W. Shockley invented transistor. The term transistor
was given by John R. Pierce. As we will go through the topic, we will know about the
transistor, mainly bipolar junction transistor or BJT. The word “transistor” is derived from
the words “Transfer” & “Resistor” it describes the operation of a BJT i.e. the transfer of an
input signal from a low resistance circuit to a high resistance circuit. This type of transistor
is made up of semiconductors.

The Transistor is a three terminal active device made from different semiconductor materials
that can act as either an insulator or a conductor by the application of a small signal voltage.
The transistor's ability to change between these two states enables it to have two basic
functions: "switching" (digital electronics) or "amplification" (analogue electronics).

The two basic families of transistors are bipolar junction transistors (BJTs) and field effect
Transistors (FETs). Our focus in this chapter will be on the bipolar junction transistor and
FETs for the next other chapters.

It would be difficult to overstate the importance of the transistor to electronics. Some of its
uses are:

 Amplifiers.

 Electronic switches.

 Digital logic.

 Memory circuits.

5.1. BASIC FOR BIPOLAR JUNCTION TRANSISTOR.

A bipolar junction transistor is a three terminal semiconductor device consisting of two N-


type and one P-type or NPN- junctions. BJTs also can be fabricated from two P-type and one
N-type or PNP- Junctions.

The basic diagrams of the two types of bipolar junction transistors mentioned above are
given below.

Figure5.1: Bipolar Junction Transistor.

48
From the above figure, we can see that every BJT has three parts named emitter, base and
collector. JE and JC represent junction of emitter and junction of collector respectively.

Figure5.2.Typical Bipolar Transistor electric symbol.

5.2. BIPOLAR TRANSISTOR CONSTRUCTION

The Bipolar transistor is obtained by joining together two individual signal back- to- back
resulting in two p-n junctions connected in series that share either a common p-type
terminal or a common n-type terminal. The resulting transistor will be named NPN or PNP
depending on which terminal is common to both P-N junctions.

Fig.5.3: Circuit Symbols of Transistors.


We must notice that the BJTs are formed from three doped region on either silicon or
germanium and the three regions are known as the Emitter (E), the base (B) and the collector
(C).

 Emitter:

It forms the left-hand section (region) of the transistor and it is more heavily doped
than any of the other regions because its main function is to supply majority charge
carries (either electrons or holes) to the base. The term bipolar returns to the fact

49
that holes and electrons participate in the injection process into the opositly polarized
material.

 Base

It forms the middle section of transistor. It is very thin (about10-6m) and lightly
doped compared to the emitter and collector regions. This lower doping level
decreases the conductivity of the material by limiting the number of free carriers in
this region.

 Collector

It forms the right-hand side section(region) of the transistor. Its main function ( as it
is indicated by its name) is to collect majority charge carriers throuhg the base.

Transistor Biasing

Depending on the bias condition (forward or reverse) of each of the two junctions making a
transistor, there are four different modes of operation of the BJT:

Mode Emitter-Base Junction Collector -Base Junction

Cut-off Reverse Reverse

Active Forward Reverse

Saturation Forward Forward

Reverse active Reverse Forward

 Active: is the one which is mostly used if the transistor is used as an amplifier

 Saturation : Equivalent to an on state when transistor is used

 Cutoff : Equivalent to an off state when transistor is used as a switch

 Inverted or Reverse active : Rarely if ever used.

NB: For proper working of a transistor it is essential to apply active mode. A transistor will
never conduct any current if its emitter-base junction is not forward biased. Therfore, in this
case, there would be no current due to majority charge carries. However, they would be an
extremely small current due to minority charge carriers which is called leakage current of the
transistors.

5.3 . NPN transistor in the active mode.


For the transistor to be operated in active mode, the emitter base juction must be forward
biased and the collector base junction reverse biased. Two external batterries are used to
establish the required bias conditions:

 The base emitter voltage VBE causes the base to be at a higher potential than the
emitter, thus forward-biasing the emitter base junction.

50
 While the collector base voltage VCB causes the collector to be higher in potential than
the base, thus reverse biasing the collector base junction.

Fig.5.4: Transistor’s current and voltages


Since the emitter base junction is forward biased, its depletion region is narrow, hence it
provides a small resistance to the flow of electrons from the emeitter region to the base. The
collector base junction is reverse biased and hence its depletion layer is very wide and
therefore offers a high resistance to the flow of majority carriers from the base to the
collector.

The negative terminal of the battery provides electrons into the emitter region. Those
electrons are majority carriers in this region and can easly diffuse into the narrow base
through the emitter base junction which is forward based and therefore offering a small
resistance. The base contains holes as majority carries and some holes and electrons
combine in the base region. Since the base is thin and lightly doped the probability of
electron hole combination in the base is very small (2%). The remaining electrons which are
minority carries in the base region cross into the collector region through the high resistance
of the depletion layer of reverse biased collector base junction and enter into the positive
terminal of the battery.

Referring to the above figure (figure 5.4b), the flow of electron from emitter to the base
produces a conventional emitter current IE in the opposite direction. Similarly the
recombination of electron and holes in the base region produces a small base current IB
which reduces slightly the collector current IC produced by the flow of electrons from the
base to the collector. The emitter current is the sum of collector and base currents.

I E  I B  IC (5.1)

IB is very small compared to IC and IE. The capital letter subscripts indicate DC values.

In similar way; the voltage VCE is equal to the sum of VCB and VBE

VCE = VCB + VBE (5.2)

5.4. Transistor circuit configuration

Basically , there are three types of circuit connections( called configurations) for operating a
transistor. These are common base, common emitter and common collector
configurations. The term common is used to denote the electrode that is common to both
input and output circuits. Because the common electrode is generally grounded, these

51
modes of operation are frequently referred to as grounded-base, grounded -emitter and
grounded -collector.

As the bipolar transistor is a 3 terminal device, there are basically 3 possible ways of
connecting it within an electronic circuit with one terminal being common to both the input
and output circuit. Each one of these configurations responding differently to its input signal
within a circuit as the static characteristic of the transistor varies with each circuit
arrangement.

5.4.1.Common base configuration.


a) Definition
In this circuit the emitter terminal of the transistor serves as the input, the collector the
output, and the base is connected to ground, or common to both. The figure below shows a
typical common base amplifier circuit. The base is the common terminal and is grounded.

Fig.5.7: Typical common base amplifier circuit.


In this configuration, emitter current IE is the input current and collecter current Ic is the
output current. The input signal is applied between the emitter and base whereas output is
taken out from the collector and base as shown in figure above.

b) Current gain ratio


 dc  .

The current gain in common base configuration is



dc and is called the “forward current

transfer ratio” for common base configuration. We should notice that the input current is IE
and the output current is IC.

IC
 dc  (5.11)
IE

Typical values of
 dc range from 0.95 to 0.99 or greater but dc is always less than 1. The

reason is that IC is always slightly less than IE by the amount of IB.

I C  I E
(5.12)

I B  I E  I C  I E  I E
 1   I E (5.13)

52
Example1

If for the transistor shown in figure 5.7b.   0.95 and I E  1mA find the values of
I C and I B
.

Solution

I C  0.95mA and I B  0.05mA

Example2

Following the current readings are obtained in transistor circuit of the same figure 5.7b.
I E  2mA and I B  20 m . Compute the value of  and I C .

Solution

I C  1.95mA and   0.99

c) Common Base Characteristics

 Input Characteristics.

It shows how IE varies with VBE when voltage VCB is held constant. For p – n – p transistor,
the input current is the emitter current (IE) and the input voltage is the collector base voltage
(VCB). The figure below shows how this variation will be by adjusting VBE while VCB remain
constant.

As the emitter – base junction is forward biased, therefore the graph of I E Vs VEB is similar to
the forward characteristics of a p – n diode. IE increases for fixed VEB when VCB increases.

 Output Characteristics

53
The output characteristics shows the relation between output voltage VCB and output current
IC. The figure below shows the output characteristics for a p – n – p transistor in CB mode.

As we know for p – n – p transistors IE and VEB are positive and IC, IB, VCB are negative. These
are three regions in the curve, active region saturation region and the cut off region. The
active region is the region where the transistor operates normally. Here the emitter junction
is reverse biased. Now the saturation region is the region where both the emitter collector
junctions are forward biased. And finally the cut off region is the region where both emitter
and the collector junctions are reverse biased.

d) Common base formulas

These two common base circuit above are equivalent. Consider the loop MEBM. In fig.
below,

By applying Kirchhoff's voltage law and starting from point B (or ground) upwards, we get

Since, generally, VEE » VBE, we can simplify the above equation to


I E  VEE / RE  10V / 20 K  0.5mA

Taking VBE into account and assuming silicon transistor, we get:

54
Example1: In the circuit below, what the value of RL causes VCB=5V?

Solution

VEE 10V
IE    1mA I C  I E  I E  1mA
RE 10 K

VCC  VCB 20  5
Now VCC  I C RL  VCB RL    15 K
IC 1mA

Example2: In the circuit below; what the value of RE causes VBC=10V?

55
Solution

VCC  VBC 20V  10V IC


IC    0.5mA Now IE   I C  0.5mA
RL 20 K 

If we neglect VBE, the entire VEE=20V has to be dropped across RE.

20V
0.5 RE  20V or RE   40 K
0.5mA

e) Common base voltage gain.

Vout I C  Rout I C  RC
AV   
Vi n I E  Rin I E  RE

The common base circuit is generally only used in single stage amplifier circuits such as
microphone pre-amplifier or radio frequency (Rf) amplifiers due to its very good high
frequency response.

5.4.2.The Common Emitter Configuration.


a) Definition
In the Common Emitter or grounded emitter configuration, the input signal is applied
between the base and emitter , while the output is between the collector and the emitter as
shown figure below. This type of configuration is the most commonly used circuit for transistor
based amplifiers and which represents the “normal” method of bipolar transistor connection.

The common emitter amplifier configuration produces the highest current and power gain of
all the three bipolar transistor configurations. This is mainly because the input impedance is
LOW as it is connected to a forward biased PN-junction, while the output impedance is HIGH
as it is taken from a reverse biased PN-junction.

b) The Common Emitter Amplifier Circuit.

56
In this type of configuration, the current flowing out of the transistor must be equal to the
currents flowing into the transistor as the emitter current is given as

IE = IC + IB.

As the load resistance ( RL ) is connected in series with the collector, the current gain of the
common emitter transistor configuration is quite large as it is the ratio of:

IC
 dc 
IB

A transistors current gain is given by the Greek symbol of Beta, ( β ).

As the emitter current for a common emitter configuration is defined as IE = IC + IB, the ratio
of IC/IE is called Alpha, given the Greek symbol of α. Note: that the value of Alpha will always
be less than unity.

Since the electrical relationship between these three currents, I B, IC and IE is determined by
the physical construction of the transistor itself, any small change in the base current ( IB ),
will result in a much larger change in the collector current ( IC ).

Then, small changes in current flowing in the base will thus control the current in the
emitter-collector circuit. Typically, Beta has a value between 20 and 200 for most general
purpose transistors. By combining the expressions for both Alpha, α and Beta, β the
mathematical relationship between these parameters and therefore the current gain of the
transistor can be given as:

IC
 dc  (5.13)
IB

IC
Recalling that  dc  and I E  I B  I C , we can get the relationship between  dc and  dc
IE

 dc
 dc  (5.14)
1   dcB

or

 dc
 dc  (5.15)
1   dcB

57
and

Where: “IC” is the current flowing into the collector terminal, “IB” is the current flowing into
the base terminal and “IE” is the current flowing out of the emitter terminal.

Then to summarise a little. This type of bipolar transistor configuration has a greater input
impedance, current and power gain than that of the common base configuration but its
voltage gain is much lower. The common emitter configuration is an inverting amplifier
circuit. This means that the resulting output signal is 180° “out-of-phase” with the input
voltage signal.

 c) Common emitter transistor characteristics

Using a circuit like shown in the figure below, we can generate a set of collector
characteristics curves for input and output.

 Input characteristics

IB (Base Current) is the input current, VBE (Base – Emitter Voltage) is the input voltage
for CE (Common Emitter) mode. So, the input characteristics for CE mode will be the
relation between IB and VBE with VCE as parameter. The characteristics are shown
below.

The typical CE input characteristics are similar to that of a forward biased of p – n diode.
But as VCB increases the base width decreases.

 Output characteristics

It shows how the collector current, IC, varies with the collector to emitter voltage, VCE, for
specified values of the basic current, IB. The biasing power supplies are variable sources
of voltage.

58
Fig.5.9: Collector characteristics of a common emitter configuration
It can be noticed that when reaches a sufficiently high voltage the reverse biased base
collector junction goes into breakdown, and the collector current increases rapidly. A
transistor should never be operated in this region.

d) Common emitter Formulas.


When we wish to design a transistor as an amplifier, we must make a dc analysis in order to determine
the appropriate value of VCC , VBB , RB and RC . We can illustrate this analysis by taking an example on
the following common emitter connection.

Fig.5.10: Typical common emitter amplifier( these two circuits are equivalent)
The voltage between the base and the emitter VBE is equal to the potential barrier of the base
emitter junction. For a silicon transistor this value is taken to be VBE  0.7 V while for
Germanium transistors VBE  0.2 V .

In the input circuit above ( according to Kirchhoff's rules):

VBB  I B RB  VBE (5.16)

VBB  VBE
 RB  (5.17)
IB

In the output circuit ( according to Kirchhoff's rules)

VCC  I C RC  VCE (5.18)

59
VCC  VCE
RC  (5.19)
IC

However we must recall that:

I C   dc I B (5.20)

VBC  VCE  VBE (5.21)

Example1

For the circuit below,

Find : (i)IB ,(ii) IC , (iii) and VCE . Neglect VBE.

Solution

VBB 10
(i) I B    10 A
RB 1M

(ii) I C  I B  100  10 A  1mA

(iii) I E  I B  I C  1mA  10 A  10.01mA

(iv) VCE  VCC  I C RC  15  1  10   5V

Example2.

Consider the circuit below and calculate the transistor currents and voltage

60
Take VBB = 10V, VCC = 20V,  dc  200 , RE = 10 k  .

Answer

V BB  V BE 10V  0.7V
IE    0.93 mA
RE 10 k 

IE 0.93 mA
Buts as I E  I C  0.93 mA ; I B ~
 
 dC 200

I B  4.65 A

 I B  4,65 mA

VC VCC  20 V ; VB VBB 10 V

V E  I E R E  0.93 m A 10 k  = 9.3 V

VCE  VC  VE  20  9.3  10.7V

5.4.3. Common collector configuration.


a) Definition
A common-collector amplifier is one of three basic single-stage bipolar junction transistor
(BJT) . In this circuit the base terminal of the transistor serves as the input, the emitter is
the output, and the collector is common to both (for example, it may be tied to ground
reference or a power supply), hence its name.
The figure below shows a common collector configuration. The input signal is applied to the
base, and the output is taken from the emitter. The common collector amplifier is usually
referred as the emitter follower since the output voltage at the emitter follows the input
voltage at the base. The main purpose of the common collector configuration is to connect a
source having a large resistance to a load with a relatively low resistance.

Fig.5.11: Typical common collector configuration.


b) The dc analysis of common collector configuration.

61
In this configuration the input current is IB whereas the output current is IE. Hence the
current gain of the circuit is given as:

IE
Ai    dc  1 (5.22)
IB

 The easy way of connecting thee CC configuration is given by the figure5.11. From this
figure , by using the Kirchhoff's rule:

 The left side loop gives us:

VBB  VBE  RB I B

or

VB VBB  VBE
IB   (5.23)
RB RB

 The right side loop gives us:

 VCE  RE I E  VCC  0

Which give:

VCE  VCC  VE  VCC  I E RE (5.24)

The common collector, or emitter follower configuration is very useful for impedance
matching applications (i.e. to maximize the power transfer ) because of the very high input
impedance, in the region of hundreds of thousands of Ohms while having a relatively low
output impedance.

 The CC circuit with its proper d.c. biasing voltage sources is shown in Fig. below (a). But
there is another way of drawing a common collector configuration; see figure below (b).
The two circuits given represent the same thing. For the second circuit (b), we have
resistances on Emitter and on base and no resistance on collector. This put the collector
terminal to serve as common , the base as the input and emitter as output . I.e. both are
common collector circuit.

Fig.5.12:Common collector Different types.


Another way of drawing the same circuit is shown in Fig. below (a) where only one battery
has been used. It should be noted that load resistor is not in the collector lead but in the
emitter lead as shown.

62
(a) (b)

Fig.5.13:Common collector with one battery.

The figure (b) above makes the circuit connection quite clear. Input is between base and
collector terminals where as output is between emitter and collector terminals.

It is seen that from the left side loop(i.e. CBE loop):

VCC  I B RB  I E RE  VBE

IC IE
As I E  I C  I B or IB  
 

 R 
VCC  VBE  I E  RE  B 
  

or

In the same way we can also calculate IB again from above equation:

VCC  VBE  I B RB  I B RE
 VBE  I B RB  RE 

Which gives

and form the right side loop(C ,IC,E loop), form Kirchhoff's rule we get:

Example: In the circuit below ; find; (a) IB (b) IE (c) VCE (d) VE and (e) VB

63
Take   49 and VBE  0.7V .

Solution.

VCC  VBE 9.0  0.7 8.3


(a) I B     41.9A .
RB  RE 100  49  2 198 K

(b) I E  1   I B  50  41.5  2.075 mA

(c) VCE  VCC  I E RE  5.85V

(d) VE  VBE  I E RE  0.7V  4.15V  4.85V

Example2 : Find the transistors currents in the circuit below ;

Solution.

Neglecting VBE;

VCC 10
IB    0.05mA
RB  RE 100  200  0.5

I C  I B  200  0.05  10mA

64
I E  I B  I C  10mA  0.05mA  10.05mA

c) Common Collector Static Characteristics .

Fig.5.14:Common collector statistics.

In this case, collector terminal is common carrier to both the input (CB) and output (CE)
carrier’s circuits. The output characteristic is IE versus VCE for several fixed values of IB.
Since IC ≈ IE, as shown in Fig. below (a). The CC input characteristic is a plot of VCB versus IB
for different values of VCE and is shown in fig. below (b).

Fig.5.14:Common collector statistics characteristics.

For IB = 100 μA and VCE = 2 V. VCB = VCE – VBE = 2 – 0.7 = 1.3 V — for Si material

Moreover, as VCB is increased, VBE is reduced there by reducing IB. Now, consider the values

VCE = 4 V and IB = 100 μA . VCB = 4 – 0.7 = 3.3 V

Again, as VCB increases, IB is decreased.

d) Relationships between DC currents and gain.

65
EXERCISES

QUESTION1.

SOLUTION

66
67
CHAPTER VI. LOAD LINES AND DC BIAS CIRCUITS.

6.1. D.C. Load Line.

A transistor must be properly biased with a DC voltage in order to operate as an amplifier.


The load line shows the collector current IC versus collector voltage VCE for different values of
base current Ibase. The intersections of the load line with the transistor characteristic curve
represent the different values of IC and VCE at different base currents.

The point on the load line where it intersects the collector current axis is referred to
as saturation point. At this point, the transistor current is maximum and voltage across
collector is minimum.

The cut-off point is the point where the load line intersects with the collector voltage axis.
Here the transistor current is minimum (approximately zero).

The dc load line is the locus of IC and VCE at which BJT remains in active region i.e. it
represents all the possible combinations of IC and VCE for a given amplifier.

For drawing the dc load line of a transistor, one needs to know only its cut-off and saturation
points. It is a straight line jointing these two points. The goal is to set the Q-point such that
it does not go into saturation or cut-off when an a AC signal is applied.

For example, take the transistor given by the figure below.

Fig.6.1. An example of a common emitter.

68
For this circuit, the voltage equation of the collector-emitter is:

Obviously, load line can be drawn if only VCC and RL are known. At saturation point B, the
maximum current is passing through the transistor. The above equation

It is a linear equation similar to y  mx  c . The graph of this equation is a straight line
whose slope is m  1 / RL .

For the CE circuit of Figure 6.1 above , the load line is drawn in Figure 6.2 below

Figure6.2: Load line and Q-point.

A is the cut-off point and B is the saturation point. From loop(VCC ,IC and VCE).

Active Region

All operating points (like C, D, E etc. in Fig. above) lying between cut-off and saturation
points forms the active region of the transistor. In this region, E/B junction is forward-
biased and C/B junction is reverse-biased - condition necessary for proper operation of a
transistor.

Quiescent or operating point

It is a point on the dc load line, which represents the values of IC and VCE that exist in a
transistor circuit when no input signal is applied.

The best position for this point is midway between cut-off and saturation points where VCE=
½ VCC (i.e. approximately in the middle of the load line and close to the saturation point, like
point D in Fig. above).

Example1.

69
For the circuit shown in Figure below, draw the dc load line and locate its Quiescent or dc
working point.

Solution

 The Load line

For the cut-off point (IC =0), from this condition we have found that:

VCE=VCC=20V ----------- point A

in figure(b) above. At saturation point B (VCE=0), saturation value of collector current is

VCC 20
I C ( sat )    4mA ---------- point B
RL 5 K

 The Q-point is located at IC and VCE point.

Therefore from Kirchhoff's rules( left side loop) we get:

 VEE  I E RE  VBE  0 by neglecting VBE

I E  VEE / RE  30 / 15 K  2mA ,

I C  I E  I E  2mA

Again; from Kirchhoff's rules (right loop) we get:

VCC  I C RL  VCE  VBE  0 by neglecting VBE

70
 VCE  VCC  I C RL  20  2  5  10V or VCE  10V

The line AB represents the load line for the given circuit. We will now find the actual
operating point.

Hence, the Q-point is located at (10V, 2mA) as it is shown in figure (b) above.

Q-Point and Maximum Undistorted Output.

Position of the Q-point on the dc load line determines the maximum signal that we can get
from the circuit before clipping occurs. Consider the cases shown in Fig. below.

Figure6.3: Q-Point and Maximum Undistorted Output.

(a) Q1 is located near cut-off point, signal first starts to clip at A. It is called cut-off clipping
because the positive swing of the signal drives the transistor to cut-off.

Maximum positive swing is  I CQ Rac .

(b) Q2 is located near saturation point, then clipping first starts at point B . It is caused by
saturation.

The maximum negative swing  VC EQ .

(c) Q3 is located at the centre of the load line. In this condition, we get the maximum
possible output signal. The point Q3 gives the optimum Q-point.

The maximum undistorted signal  2VECQ .

In general, consider the case shown in Fig. below.

71
Figure 6.4: maximum undistorted signal

Since A < B, maximum possible peak-to-peak output signal = 2 A. If the operating point
were so located that A > B, then maximum possible peak-to-peak output signal = 2B. When
operating point is located at the centre of the load line, then maximum undistorted peak-to-
peak signal is = 2 A= 2 B= VCC= 2 VCEQ.

Under optimum working conditions corresponding to Fig. 6.3 (c), ICQ is half the saturation
value given by VCC/RL.

Example1.

In the CB circuit fig (a) below, find

(a) dc operating point and dc load line.

(b) maximum peak-to-peak unclipped signal.

Solution

(a) I C ( sat )  VCC / RL  20 / 10 K  2mA --------- point B.

 VCE ( at cutoff )  VCC  20V -------- point A.

Hence, AB is the dc load line and is shown in figure below.

72
Now

I E  10 / 20  0.5mA and I C  I E  0.5mA .

 VCE  VCC  I C RL  20  0.5  10  15V by neglecting VBE

The Q-point is located at (15V, 0.5mA)

(b) It is obvious from fig(b) above, that the maximum positive swing can be from 15V to 20 V
i.e. 5V only. Of course , on the negative swing, the output swing can go from 15V down to
zero volt. The limiting factor being cut-off on positive half -cycle, hence maximum unclipped
peak-to-peak voltage that we can get from this circuit is 2x5=10V.

Example2.

Determine the value of RB required to adjust the circuit below to optimum operating point.
Take Beta=50 and VBE=0.7V.

Solution

As seen from above:

73
20
I CQ   1mA
2  10

The corresponding base current is:

I CQ 1
I BQ    1 / 50  20 A
 

Now VCC  I B RB  VBE

VCC  VBE 20  0.7


RB    965 K
IB 20  10 6

Proper and improper dc biasing figures:

The goal of amplification in most cases is to increase the amplitude of an ac signal without
altering it. The following figures show the effects of proper and improper dc biasing of an
inverting amplifier.

Fig 6.5(a): dc biasing.

The region along the load line including all points between saturation and cut off is generally
known as the linear region of the transistor’s operation. As long as the transistor is operated
in this region, the output voltage is ideally a linear reproduction of the input signal.

74
Figure 6.5 (b) dc biasing.

6.3. DIFFERENT METHODS OF BIASING A TRANSISTOR.

In this section we will study the different ways to bias a transistor. Remember that to properly
polarize a transistor for normal operation, you must use a voltage source VBB required to
forward bias the EB junction. In addition, you must use a source voltage VCC to make the CB
reverse biased junction. In this section you will learn other ways to achieve this condition.

6.3.1.Base bias.

a) Base bias or fixed current bias.

Base bias is the simplest way to bias a BJT transistor. Base bias ensures that the voltage fed to
the base. It is possible to use only one source of a battery, namely VCC, instead of two for
biasing the transistor. This technique is illustrated in Figure below. Note that the two circuits
below are equivalent.

75
Fig.6.1:) The base bias
Base Bias Voltage/Current Calculations

In this type of transistor bias we normaly use the following important formulas.

 For the left side loop we have:

VCC  VBE  I B RB (6.1)

Which can give the following equation:

VCC  VBE
IB  (6.2)
RB

 For the right side loop we have:

VCC  VCE  I C RC (6.3)

or

VCE  VCC  I C RC (6.4)

and knowing that I C   dc I B , we get

VCE  VCC   dc I B RC (6.5)

Disadvantages of Base Bias Method

Though base bias is one of the simplest and easiest methods to bias transistors, it is the least
popular way to do so. This is because the collector current, I C, is decided by purely by the β of
the transistor.

β of a transistor is one of the most unstable and unpredictable parameters of a transistor. β


can vary largely across transistors( i.e. 20-200) even of the same exact model and type.
Therefore, base bias can lead to unpredictable actions if a transistor needs to be replaced and
there are variations in the β of that transistor. β is also susceptible to changes due to
temperature, as it can vary pretty largely due to ambient temperature. Base bias, then, can
produce erratic circuit behaviour due to transistor variations.

76
This method is not a very satisfactory method because bias voltages and currents do not
remain constant during transistor operation. We will explore other methods of transistor
biasing including voltage-divider bias and emitter-supply bias, which provide more stability
against the changes that may exist in β of a transistor.

b) Base bias with emitter feedback.

This circuit achieves good stability of dc operating point against changes in β with the help of
emitter resistor which causes degeneration to take place.

Figure 6.2: Base bias with emitter feedback.

This circuit is obtained by simply adding an emitter resistor to the base bias circuit as shown in
Fig. below.

Current and voltage calculations for Base bias with emitter feedback.

 From the figure above , the Kirchhoff's rules gives us:

VCC  I C RC  I E RL  VCE (6.6)


At saturation, VCE is essentially zero; hence VCC is distributed over RL and RE.

VCC
I C ( sat )  (6.7)
RE  RL

 IC can be found by applying Kirchhoff’s Voltage Law, considering the supply, base,
emitter and ground route. we have

VCC  I B RB  VBE  I E RE  0 or VCC  I B RB  VBE  I E RE (6.8)

Now

IC
IB  and I E  I C

Substituting these values in the above equation, we have

77
IC
VCC  RB  VBE  I C RE

VCC  VBE VCC


IC   (6.9)
R R
RE  B RE  B
 

Neglecting V BE .

 Collector-to-ground voltage

VC  VCC  I C RL (6.10)

 Emitter-to-ground voltage

VE  I E RE  I C RE

The beta sensitivity for this circuit is

1
  (6.11)
R
1 E
RB

N.B: The beta sensitivity of a circuit is meant the influence that the beta-value has on its dc
operating point. Variation in beta-value are caused either by variations in the circuit operating
conditions or by the substitution of one transistor with another. The beta sensitivity is a
dimensionless ratio and can range from zero to unity.

Example:

For the circuit shown in Fig. below,

find (i) IC (sat) , (ii) IC , (iii) VC , (iv) VE, and (v) VCE (vi)  

Solution.

VCC 30
(i) I C ( sat )    10mA
RE  RL 1  2

78
VCC 30
(ii) I C    7.5mA
RB 300
RE  1
 100

(iii) VC  VCC  I C RL  30  2  7.5  15V

(iv) VE  I E RE  I C RE  7.5  1  7.5V

(v) VCE  VC  VE  15  7.5  7.5V

1
(vi)    7 .5 .
1  100  1 / 300 

c. Base bias with collector feedback.

It is also known as collector-to-base bias or collector feedback bias. It provides better bias
stability.

Figure6.3: Base bias with collector feedback.

This circuit (Fig. below) is like the base bias circuit except that base resistor is returned to
collector rather than to the VCC supply. It derives its name from the fact that since voltage for
RB is derived from collector, there exists a negative feed-back effect which tends to stabilise IC
against changes in β. To understand this action, suppose that somehow β increases. It will
increase IC as well as IC RL but decrease VC which is applied across RB. Consequently, IB will be
decreased which will partially compensate for the original increase in β.

VCC
I C ( sat )  (since VCE  0 for saturation).
RL

VC  VCC  I B  I C RL  VCC  I C RL (6.12)

Also

79
VC  I B RB  VBE

Equating the 2 expressions of VC, we get:

I B RB  VBE  VCC  I C RL (6.13)

Since

IC
IB  we get

IC
 RB  VBE  VCC  I C RL

VCC  VBE VCC


IC   (6.14)
RB R
RL  RL  B
 

This is the approximate value of IE. Again , by the help of beta rule, thus the beta sensitivity for
this circuit is:

1 I
  1 C (6.15)
1  RL / RB I C ( sat )

Example

For the circuit below compute (a) the three transistor current (b) drop across RC and RB (c)
value of VCE and (d) beta sensitivity   .

Solution

12  0.7
(a) IC   56.5mA
100  12 K / 102 

80
I B  I C /   56.5mA / 120  471A

I E  I C  I B  57 mA

(b)

I RC  I C  I B  I E  57 mA

VRC  I RC RC  57 mA  100  5.7V

VRB  I B RB  471A  12 K  5.67V

(c)

VCE  VCC  VRC  12  5.7  6.3V

1 1
(d)     0.54 .
1  RL / RB 1  100  0.1K  / 12 K

d) Base Bias with Collector and Emitter Feedbacks

In the circuit of Fig. below:

Figure6.4: Base Bias with Collector and Emitter Feedbacks.

Both collector and emitter feedbacks have been used in an attempt to reduce circuit sensitivity
to changes in β. If β increases, emitter voltage increases but collector voltage decreases. It
means that voltage across RB is reduced causing IB to decrease thereby partially off-setting the
increase in β. Under saturation conditions, VCC is distributed over RL and RE. Assuming IB to
be negligible as compared to IC, we get;

IC(sat) = VCC/(RE + RL). (6.16)

81
Actual value of I C is equal to:

VCC  VBE
IC  (6.17)
R
RE  RL  B

going via RB because VCE is unknown

VC  VCC  I B I C RL  VCC  I C RL

VE  I E RE  I C RE VCE  VC  VE (6.18)

VCE  VCC  I C RL  RE 

It can be proven that

1 I
   1 C (6.19)
1   R E  RL  / R B I C ( sat )

Obviously ,   will be degrade with increase in RB.

Example: For the circuit below,

Calculate:

(i) IC (sat) , (ii) VCE and (iii)   . neglect VBE and take beta=100.

Solution

82
VCC 15
(i) I C ( sat )    0.75mA
RE  RL 10  10

(ii) VCE  VCC  I C RL  RE  VCC 15


and I C    0.6mA
RE  RL  RB /  10  10  500 / 100

Therefore;

VCE  15  0.6(10  10)  3V

1 1 I 0.6
(iii)     0.2 or    1  C  1   0.2
1   RE  RL  / RB 1  100(10  10) / 50 I C ( sat ) 0.75

Obviously ,   will be degrade with increase in RB.

e. Voltage divider bias.

It is most widely used in linear discrete circuits because it provides good bias stability. It is also
called universal bias circuit or base bias with one supply.

Figure 6.5:Voltage divider bias.

This arrangement is used for transistor incorporated in integrated circuits(ICs). The name
'voltage divider' is derived from the fact that the resistors R1 and R2 form a potential divider
across VCC. (see figure below).

83
Fig. 6.6: The voltage divider bias circuit a) complete circuit b) approximation for low IB.

The voltage drop across R2 forward biases the emitter whereas VCC supply reverse biases the
collector. Considering the approximation for low IB figure, the current passing through R1 is
equal to current passing through R2. Therefore according to Ampere's law this current is given
by

VCC
I (6.20)
R1  R2

In similar way by using Ampere's law, we can calculate the voltage for resistance R2 as:

VCC VCC
VR 2   R2 or VR1   R1 (6.21)
R1  R2 R1  R2

The above equation is known as voltage divider theorem.

Using Kirchhoff' rule (down loop), from the figure above give, we get:

I 2 R2  VBE  I E RE  0

or

VE  VR 2  VBE (6.22)

VE VR 2  VBE VR 2
As IE    (6.23)
RE RE RE

Considering the figure above (right side: the loop from VCC-RC-VCE to RE)

VCC  I C RC  VCE  I E RE  0

we see that from the figure above

VC  VCE  I E RE

84
thus the voltage for collector terminal is given by:

VC  VCC  I C RC (6.24)

And finally,

VCE  Vc  VE  VCC  I C RC  I E RE
(as I C  I E ) (6.25)
 VCC  I C ( RC  RE )

As before

VCC
I C ( sat )  (for saturation VCE  0 )
RE  RL

VCE ( cut off )  VCC (for cut off I C  0 )

The beta sensitivity is equal to:

1
 

1   RE / R1 R2  (6.26)

It is seen from above calculations the value of beta was never used anywhere. The base voltage
is set by VCC and R1 and R2. The dc bias circuit is independent of transistor  . That is why it

is such a very popular bias circuit.

Example:

For the circuit below,

Calculate:

(i) IC (sat) , (ii)IC (iii) VCE and (iv)  . neglect VBE and take beta=50.

85
Solution

VCC 20
(i) I C ( sat )    2.5mA
RE  RL 2  6

VR 2 6
(ii) I C  I E    1mA
RE 6

(iii) VCE  VCC  I C RL  RE   20  1  (2  6)  12V

(iv) R 1 
R2  84 / 20  4.2 K

1 1
    0.0138 .
1   RE / R1 R2 
1  50  6 / 4.2

6.3.2. Emitter bias.

One way to get stabilized condition in transistor biasing is to use the emitter bias . In emitter
bias the base voltage is approximately zero and it is used two batteries one positive and
another negative. The circuit gets this name because the negative supply VEE is used to forward
bias the emitter junction through resistor RE. VCC still reverse biases collector junction. This
also gives the same stability as voltage divider circuit but it is used only if split supply is
available. These are illustrated in the circuit below:

Fig.6.7: The emitter bias circuit.


In this type of transistor bias we normally use the following important formulas:

VB  0 or I B R B  0 (6.27)

We have three branches one for base, other for emitter and another for collector.

 For the branch of emitter we have:

 I B RB  VBE  I E RE  VEE  0 (6.28)

From (27) we get:

86
VEE  VBE
IE  (6.29)
RE

Also From approximation we have:

I E  IC (6.30)

 For the branch of VCC through VCE to VEE loop we get:

VCC  I C RC V CE I E RE  VEE  0 (6.31)

Putting (30) into (31) we get

VCC  I C RC V CEVEE  VBE   VEE  0 (6.32)

By neglecting VBE in this equation we get:

VCE V CC I C RC (6.33)

From voltage theorem we know that:

VCE V CVE (6.34)

The stability of the emiter bias

We can show the stability of the emiter bias ; by considering equation 6.28;

 I B RB  VBE  I E RE  VEE  0

but

IE
IB  (6.36)
 dc

R 
 I E  B  RE   VEE  VBE
  dc 

VEE  VBE
IE  (6.37)
R
RE  B
 dc

RB
Since RE  , we get
 dc

VEE  VBE
IE   IC (6.38)
RE

which is independent of β and therefore more stable. Assuming that V EE  V BE we obtain

87
VEE
IE  (6.39)
RE

This show that the emitter current doesn't depend on βdc and V BE when the above conditions
are satisfied. Therefore the emitter bias will not varies with the parameter βdc and V BE thus,
the Q-point of the amplifier will be very stable.

Example

Find I E , I C and VCE in figure below:

where RB  50k, RC  1k, and RE  5k

Answer

VE  VBE  0.7V

VE  VEE 0.7V  (10V ) 9.3V


IE     1.86mA
RE 5k 5k

 I E  1.86 mA

I C  I E  1.86mA

VC  VCC  I C RC  10V  (1.86mA)  (1k)   8.14V


VCE  VC  VE  8.14V  (0.7V )  8.84V

 VCE  8.84V

88
6.4. Transistor equivalent circuit.

To visualize the operation of a transistor in an amplifier circuit, it is often useful to represent


the device by an equivalent circuit. Under this part we analyse the behaviour of a transistor
when a small input signal is applied. Small input signal voltage and current are in order of
10per cent of Q-point voltages and current.

6.4.1. Equivalent circuits for a CB Amplifier.

a) DC equivalent circuit.

In an ideal transistor the emitter diode acts like any forward biased diode. However, due to
transistor action the collector diode acts as current source. In other words an ideal transistor is
nothing more than a rectifier diode in emitter and a current source in collector.

b) AC equivalent circuit.

In the case of small input AC signals, the emitter diode does not rectifier, instead it offers a
resistance named AC resistance. Therefore the emitter diode is replaced by the ac resistance
noted as rac . As usual, the collector diode acts as a current source.

Fig.6.8: The ac equivalent circuit of a transistor.


c) r-parameters

There are five r-parameters making the equivalent circuit for a bipolar junction transistor
working as an amplifier.

r-parameter description

 ac AC alpha

 ac AC beta

re AC emitter resistance

89
rb AC base resistance

rc AC collector resistance

The effect of the ac base resistance( rb ) is usually very small enough to be neglected, so it is

usually replaced by a short circuit. The ac collector resistance ( rc ) is usually several hundred

kilo Ohms and can be replaced by an open circuit. For amplification analysis the ac emitter

resistance ( re ) is the most important of the r-parameter.

To get the AC resistance rac replacing the emitter diode ( after long calculations) we get:

25mV 25mV 25mV


rac    (6.40)
IB IC IE

or

 rac  re (6.41)

Where

25mV
re  (6.42)
IE

The equation above is the junction resistance of the emitter. In this formula IE is expressed

in mA to obtain re in ohms.

d) Drawing equivalent circuits

 For drawing the DC equivalent circuit the following procedure is adopted

i. Short all ac sources which means that they we reduces them to zero.

ii. Open all capacitors because they block the dc current.

Consider the circuit below:

90
Figure6.9: Amplifier circuit.

For drawing the DC equivalent circuit , after following the above procedures for A; we get
the equivalent circuit as the figure below:

Figure6.10: The DC equivalent circuit.

Here I C  I E and VCB  VCC  I C RC

 For drawing the AC equivalent circuit the following procedure is also adopted

i. All dc sources are shorted i.e they are treated as ac ground.


ii. All coupling capacitors are shorted because ac current passes through them easily.
iii. The emitter diode is replaced by its ac resistance.

Consider the same circuit of the figure 6.9; the AC equivalent circuit after following the
above procedures is now given by:

91
Figure6.9: The AC equivalent circuit.

To analyse the AC equivalent circuit we have to calculate the input and output values of
currents and voltages. After we calculate the currents , voltages and power gains of any given
amplifier circuit.

 Input resistance

From the AC equivalent circuit above, the input resistance is given by the sum of RE and re in
parallel:

re RE
rin  RE re 
RE  re

In practice RE is always much greater than re so that parallel combination:

RE re  re

And rin  re = input of emitter diode.

 Output resistance or AC load resistance

The output resistance or AC load resistance is given by the combination in parallel of RC and
RL.

rout  rL  RL RC

NB: In case the RL is not connected then the output resistance is rout  RC .

iC
 Current gain : It is given by Ai  
iE

92
Vout
 Voltage gain: Av 
Vin

The input voltage is given by: Vin  iE  rin  1   iB  rin .

The output voltage is given by: Vout  iC  rout  iC  rout  iB rL .

Vout iB rout r r


Now the voltage gain is given by: Av    L  L
Vin 1   iB rin rin re

 Power gain

  rL
The power gain now is : AP  Av  Ai 
re

When the power is expressed in decibel, it is written as: G P  10 log AP dB

Example 1.

For the single stage CB amplifier shown in figure below: (assume a silicon transistor with
  0.98 )

Find:

(a) rin , rL , Ai , Av and Ap ;

(b) what would be the r.m.s value of the signal voltage across the load if v s has an r.m.s of

1.5mV?

Solution

93
For finding re let us find I E .

VEE  VBE 10  0.7


(a) I E    1.86mA
RE 5K

25mA 25
re    14
IE 1.86

Therefore;

(i) rin  re RE

1 1 1 1 1
     rin  13.96  14 .
rin re RE 14 5K

rL rL 4 K
(ii) rL  RC  4 K (iii) Ai    0.98 (iv) Av     286
rin re 14

(v) Ap  Ai Av  0.98  286  280 or G p  10 log 280  24.5dB

(b) Output voltage  Av  Input voltage

VOut  Av Vin  1.5 103  286  0.429V

6.4.2. Equivalent circuit for a CE Amplifier.

Consider the CE amplifier of the following circuit and analyse it.

Fig 6.10: Circuit with small signal input.

 DC Analysis

To analyse this amplifier circuit with the dc equivalent circuit as prescribed by the adopted
procedure. The dc bias must first be determined. To do this a dc equivalent circuit is

94
developed by replacing the coupling and the bypass capacitors with open (remember that a
capacitor appears open to dc).

The dc component of the circuit sees only the part of the circuit that is within the boundaries
of the capacitors as the dc current will not pass through these components. The equivalent
circuit for dc analysis is shown below:

Fig.6.11: Equivalent circuit of Fig. 6.10 for dc analysis.


From the dc equivalent circuit below we can calculate the transistor currents and voltage
and determine the operating point.

The methods for analysis of the dc equivalent circuit are the same as dealing with a normal
common emitter circuit. As per voltage divider theorem,

Vcc  R2 15  5
VR 2    3V
R2  R1 20  5
VE  VR 2  VBE  3  0.7  2.3V

As

VE VR 2  VBE 2.3
IE   
RE RE 500
I E  4.6mA

VC  Vcc  IC RC  15  4.6  1  11.4V

And finally,

VCE  Vc  VE  VCC  I C RC  I E RE
 VCC  I C ( RC  RE )
 15  4.6(1  0.5)  8.1V

(as I E  I C ).

The operating point is VCE=8.1V and IC=4.6mA.

95
 AC analysis

Therefore; fro AC analysis the emitter diode is replaced by the ac resistance.

25mV 25mV
rac   re , where re    5.4
IE 4.6mA

rac   re  120  5.4  652  = 0.652k

The equivalent circuit of figure 6.10 for AC analysis is shown in figure below:

Fig.6.11: Equivalent circuit of Fig. 6.10 for ac analysis.


NB: The emitter resistor has been removed, and the bypass capacitor C3 id shorted as it
allow ac current passes through them easily.

 Input resistance:

On the input side three resistances becomes parallel across the source voltage Vs. The input
resistance of the stage is equal to input resistance of base rin as the resistance of the source
is zero and is given by:

rin  R1 // R2 // rac

or

1 1 1 1
   , rin  560
rin R1 R2 rac

96
Fig.6.12: Input ac resistance.

 Output resistance (Fig.6.12:b)

Ac load resistance is obtained by considering RC in parallel with RL

1  10
rL  RC / / RL   0.9k 
1  10

Fig.6.13. Output ac resistance.


 The current gain

The current gain for a common emitter is given by the ratio:

iC
Ai     120
iB
 The voltage gain

It is given by the ratio:

97
Vout
Av 
Vin
As there is no internal resistance of the source, Vin  Vs . In put voltage is equal to emitter
current times input resistance rin .

Vin  iB  rac  iB  re

Vout  iC  rL  ib rL

And from these two equations we get

Vout i r r 900
Av  = b L  L =  166 .6
Vin iB r e re 5.4

 Power gain:

Is given by

Ap  Ai  Av  120 166.66  20 000

When the power gain is expressed in decibels we have

G p  10 log10 Ap  10 log10 20000  43.03dB

Exercise 1

Consider the CE circuit for the figure below and compute the output voltage assuming a silicon
transistor.

Answer
a. We start by drawing the dc equivalent circuit and estimate the transistor currents and
voltages

98
As per potential divider theorem, we get from the dc equivalent circuit

R2 10 k
VB  Vcc  10V  2V
R1  R2 50 k

 V B  2V

VE VB  0,7V  2V  0.7V  1.3V

VE 1.3V
IE   = 1.3mA
RE1  RE 2 1k 

VCE VCC  I C RC  10V – (1.3mA)(5k  ) = 10V – 6.5 V = 3.5V

 VCE = 3.5V

b. For ac equivalent circuit.

Figure. Ac equivalent circuit.

99
In the equivalent circuit above, the emitter resistor RE2 has been removed, and the bypass
capacitor C is shorted as it allow ac current passes through them easily. and we remain only
by one emitter resistor RE1. We should notice that the second resistance at the emitter was
taken away by the by pass capacitor. The junction resistance of the emitter diode

25 mV 25
re    19.2
I E mA 1.3

The input resistance at the base

rin  R1 R2 rac

Where rac   re  RE1   160(500  19.2) = 160 x 519.2 = 83.1k 

Hence rin  40k 10k 83.1k  7.3k .

The attenuation constant due to source resistor is:

 r  Vin  ri n  7.3k 7.3


Vin  VS  i n  or       0.94
 Rs  rin  VS  Rs  rin  600   7.3k 7.8

 Attenuation = 0.94

The output resistance is given by

rl 
RC RL

5k100k = 4.8k 
RC  RL 5k  100 k

The voltage gain is given by:

rL 4.8k 4.8
Av     9.2
re  RE1 519 .2 0.5192

The total gain

Av  (0.94)(9.2)= 8.65

The output voltage

Vvout  AvVS  8.65 10mV  86.5mV

Current gain; Ai    160

100
Power gain Ap  160  8.65  1384

or

Power gain decibel G P  10 log 1384  31 .4dB

Exercise 2

In the CE amplifier circuit of figure below employing emitter feedback, find;

a) rin , rL , Ai , Av , AP and G p . Take transistor of   100 .

b) How will these value will value if emitter by pass capacitor C3 is removed?

Solution

a) By using the DC equivalent circuit below

101
VCC 30
IB    10 A
RB  RE 2M  100 10 K

I C  I B  100 10A  1mA

I E  I C  1mA

25mA 25mA
re    25
IE 1mA

rac  re  100  25  2500 

 By considering AC equivalent circuit below

Now;

i) rin  RB re  2M 2500   2500 

ii) rL  RC RL  10k 20k  6.67 k

Vout 6.67 k
iii) Av    267
Vin 25

iv) Ap  Ai  Av  100  267  26700 and G p  10 log 26700  44.26dB

b) When the bypass capacitor is removed, then the ac current is now passing through the
emitter resistor RE therefore, the equivalent circuit become;

102
The r- parameters becomes;

i) rin  RB  re  RE   RB  RE  ,

2
 2 M 1M  M  666 .7 k ....... rin is increased because RE is much greater than re.
3

ii) rL  RC RL  10k 20k  6.67 k , .................. it remain unchanged.

iii) Vin  ib  re  RE  and Vout  ic rL  ib rL

Vout ib rL rL r 6.67 k


therefore Av     L   0.667 , ......reduced drastically.
Vin ib  re  RE  re  RE  RE 10k

iv) Ap  Ai  Av  100  0.667  66.7 and G p  10 log 66.7  18.24dB , ........reduced considerably.

It is seen that by removing bypass capacitor, excessive degeneration has occurred in the
amplified circuit.

103

You might also like